Microbiology

अब Quizwiz के साथ अपने होमवर्क और परीक्षाओं को एस करें!

Pseudomonas aeruginosa is often used as a bacterial species for the quality control of Triple Sugar Iron (TSI) Agar. The expected reaction is: 1) Acid slant/acid butt 2) Alkaline slant/acid butt 3) Alkaline slant/acid butt/H2S 4) Alkaline slant/alkaline butt

Alkaline slant/alkaline butt Alkaline slant/alkaline butt is the correct answer because Pseudomonas aeruginosa is a nonfermenter; therefore, it cannot produce acid from glucose (butt) or lactose (slant) in a fermentative medium. As such, there is no change in the TSI tube and both the slant and the butt remain alkaline (red). Acid slant/acid butt is incorrect because this reaction pattern indicates the organism can ferment both glucose and lactose. Escherichia coli, a lactose fermenter, is used as the control organism to visualize the acid slant/acid butt reaction. Alkaline slant/acid butt is incorrect because this reaction pattern indicates the organism can ferment glucose only. Shigella flexneri, a non-lactose fermenter, is used to visualize the alkaline slant/acid butt reaction. Alkaline slant/acid but/H2S is incorrect because this reaction pattern indicates the organism can ferment glucose only and produce H2S. Salmonella typhimurium is used to detect the alkaline slant/black deep (production of H2S) reaction.

Parasites that belong to the category Sporozoa are also known as: 1) Flukes 2) Digenea 3) Hemoflagellates 4) Coccidia

Coccidia Coccidial parasites do not possess specific organelles term-125for locomotion and thus belong to the Sporozoa. Coccidia are a group of protozoal parasites where asexual replication occurs outside of the human host and sexual reproduction occurs inside the human host. Digenea is a class of parasites that includes the flukes. Hemoflagellates belong to the Mastigophora and are flagellates found in blood and tissue.

5% sheep blood agar will support the growth of most strains of the following organisms, EXCEPT: 1) Haemophilus spp. 2) Staphylococcus aureus 3) Bacillus anthracis 4) Listeria monocytogenes

Haemophilus spp. 5% sheep blood agar contains protoporphyrin IX, but not NAD (nicotinamide adenine dinucleotide) required by Haemophilus spp. for growth. All of the other organisms listed will grow on sheep blood agar. Haemophilus spp. requires chocolate agar to grow, as it provides hemin (X factor) and NAD (V factor).

What is the primary target of HIV? 1) Heart 2) Neutrophils 3) B-cell lymphocytes 4) Helper T-cell lymphocytes

Helper T-cell lymphocytes Helper T-cell lymphocytes (CD4 cells) is the correct answer because HIV infects helper-T cell lymphocytes. Lymphocytes are essential to the adaptive immune response. Another cells that is a target of HIV is a macrophage. Heart is incorrect because immune cells are the primary cells that are infected. Hence, the name Human Immunodeficiency Virus (HIV). Neutrophils is incorrect even though they are a type of white blood cell that plays a role in our immune response; however, they are not infected by HIV. Neutrophils primarily fight against bacterial infections. B-cell lymphocytes is incorrect even though they are lymphocytes. However, they are not susceptible to infection by HIV

From the choices listed below, choose the most likely associated bacterial species represented in the image as pink, runny, mucoid colonies at 48 hours incubation on MacConkey (MAC). 1) Haemophilus species 2) Pseudomonas aeruginosa 3) Klebsiella pneumoniae 4) Proteus species

Klebsiella pneumoniae The colonies of Klebsiella pneumoniae are characteristically mucoid from the production of abundant capsular material. K. pneumoniae is a lactose fermenter, indicated by pink coloration on MacConkey as the pH decreases. Colonies of Enterobacter may also produce this type of colony appearance. Although Haemophilus species may also produce capsular material, Haemophilus species do not grow on MAC. Enrichment agar such as CHOC is used for isolation. Pseudomonas aeruginosa is a non-fermenter that may demonstrate ß-hemolysis on blood agar, with flat, spreading colonies and a metallic sheen. Many strains of P. aeruginosa produce pigments that appear green, and strains may have a fruity odor. Colonies of Proteus species typically swarm when grown on blood agar, and do not ferment lactose. Proteus species are a common cause of urinary tract infection and tend to produce a distinctly alkaline urine pH because of strong urease activity and the release of ammonia.

Illustrated in the top photograph is the appearance of a 4-day old fugal colony cultured from a "ringworm" cutaneous infection. Although the appearance of the colony may suggest a presumptive identification, microscopic mounts prepared from the surface hyphae must be examined, as observed in the lacto-phenol blue stained tease mount in the bottom photomicrograph. With these observations, select from the multiple choices the identification of this dermatophyte. 1) Microsporum canis 2) Trichophyton mentagrophytes 3) Epidermophyton floccosum 4) Microsporum gypseum

Microsporum canis Microsporum canis is the correct selection. M. canis colonies mature in about 1 week. Distinctive is the peripheral lemon-yellow apron is seen at the growing margin where conidia are being produced. Also distinctive are the large, spindle shaped multi-celled macroconidia each with a thick echinuate wall. The terminal cell of each macroconidium is tapered and deviates to one side. Rare, scattered microconidia, each borne laterally directly from the hyphae, may be observed. M. canis is a zoophilic dermatophyte with human infections often contracted from handling an infected family pet. Trichophyton mentagrophytes colonies are either cottony or granular depending on maturation and are tan to cream colored. A peripheral yellow ring is not observed. Microscopically are observed grape-like clusters of small spherical microconidia irregularly spaced in loose clusters along thin hyphae. Macroconidia are typically cigar-shaped with a narrowing of attachment to their base. Epidermophyton floccosum colonies are silky gray-yellow or olive green to khaki colored, with outward projecting delicate hyphal strands that can be a dull orange-brown. Microscopically, numerous macroconidia that are multiseptate and typically large with and club-shaped with smooth walls are seen. Macroconidia are rounded at the tip attached directly and laterally from hyaline hyphae. Microconidia are not observed. Microsporum gypseum colonies are cinnamon-brown with outward projecting delicate hyphae. The underside is orange or brown. In stained microscopic mounts, large multi-celled canoe-shaped (ellipsoidal) macroconidia with smooth walls are observed, with the distal cells being rounded rather than with a tapered tip. This is a geophilic (free-living soil organism) that rarely causes human disease

On occasion in cases of heavy infection, rhabditiform nematode larvae, measuring up to 1.5 cm long, may be observed in mounts prepared from stool specimens. A presumptive species identification can be made by microscopically observing the buccal cavity as illustrated in the photograph. What is the most likely species of this nematode? 1) Strongyloides stercoralis 2) Enterobius vermicularis 3) Necator americanus 4) Ascaris lumbricoides

Necator americanus Necator americanus is one of the hookworms. This species can be identified by observing the rhabditiform larvae that characteristically have a long buccal cavity as illustrated in the photograph. Strongyloides stercoralis rhabditiform larvae have a short buccal cavity. On further examination, a prominent genital primordium may also be observed about one-third the distance from the tail. Enterobius vermicularis does not have an external host and rhabditiform larvae are not formed. Only adult worms and ova are observed in humans. Enterobius can be excluded from this exercise. Ascaris lumbricoideslarvae develop into adult worms within the human intestine. The adult worms measure between 15 and 35 cm and may be best recognized by their curved tail. A buccal cavity is not distinctive.

All of the following are responsibilities of sentinel laboratories within the Laboratory Response Network (LRN), EXCEPT: 1) Refer samples to LRN reference laboratory when select bioterrorism agents cannot be ruled out. 2) Maintain a BSL-2 facility. 3) Perform full identification and susceptibility on all potential bioterrorism agents. 4) Perform testing to rule out the presence of select bioterrorism agents.

Perform full identification and susceptibility on all potential bioterrorism agents. Sentinel laboratories should refer samples when select bioterrorism agents cannot be ruled out, must maintain a BSL-2 facility and should perform testing to rule out the presence of select bioterrorism agents Always follow your facilities procedures and contact your LRN reference laboratory for guidance with regard to the toxins and viruses that may be used as bioterrorism agents. A sentinel laboratory should NOT attempt to: Accept or process environmental or animal samples Culture from clinical specimens or detect for these agents Collect specimens for suspect viruses unless directed by a public health official

Which bacterial species is most likely represented by the opaque, chalky, white colony appearance as illustrated in the photograph? 1) Bacillus cereus 2) Listeria monocytogenes 3) Burkholderia pseudomallei 4) Streptomyces anulatus

Streptomyces anulatus From the bacterial species included here, the opaque, chalky, white colonies are produced by the nocardioform bacteria, Streptomyces (formerly greseus) anulatus. This species can also be suspected if the colonies have a pungent, musty basement odor. Bacillus cereus colonies are characteristically spreading, gray-white, and beta hemolytic, with a "frosted glass" appearance. Listeria monocytogenes produces small, round, translucent colonies that characteristically have a narrow zone of beta hemolysis, noted if colonies are removed. Burkholderia pseudomallei also produces spreading colonies, but more wrinkled in appearance and without beta hemolysis.

Which of the following morphological features would correspond to Phialophora verrucosum? 1) Polar germ tubes 2) Transverse and longitudinal septations 3) Urn or vase shaped phialides 4) Black yeast colony

Urn or vase shaped phialides The production of urn or vase shaped phialides is characteristic of Phialophora verrucosum. Polar germ tubes are produced by Bipolaris species. The genus name comes from the production of germ tubes extending from both sides of a conidia that has been incubated in an aqueous environment for several hours. Transverse and longitudinal septations are seen in conidia of Alternaria species. The identifying characteristic of Aureobasidium pullulans is the production of a black yeast-like colony.

In the preliminary identification of the Enterobacteriaceae, it is helpful to separate the various species into tribes. Which of the following is the biochemical characteristic for which the tribe Klebsielleae can be recognized? 1) Phenylalanine deaminase 2) + + - - IMViC reactions 3) Voges Proskauer 4) Hydrogen sulfide

Voges Proskauer The production of acetyl methyl carbinol, positive Voges Proskauer (VP), is common to all members of the Tribe Klebsielleae. Phenylalanine deaminase activity is linked to the tribe Proteeae. + + - - IMViC reactions are characteristic of the tribe Escherichieae. IMViC stands for indole, methyl red, Voges Proskauer and citrate. Hydrogen sulfide, although not unique, separates the tribe Edwardsielleae, from the Tribe Escherichieae.

Bordet-Gengou plates are used to isolate: 1) Mycobacterium tuberculosis 2) Corynebacterium diphtheriae 3) Bordetella pertussis 4) Haemophilus influenzae

Boretella pertussis Bordet-Gengou plates are used to isolate Bordetella pertussis. B. pertussis is a respiratory pathogen that lives within the epithelial cells of the respiratory tract. B. pertussis is the major causative agent of whooping cough. Bordet-Gengou plates are inoculated with a swab from a patient's posterior nasopharynx when a Bordetella bacterium is thought to be present. Colonies are small and shiny, resembling mercury drops. Colonies become whitish gray with age. Mycobacterium tuberculosis is commonly isolated by using Lowenstein-Jensen (L-J) media. Corynebacterium diphtheriae is commonly isolated by using either cystine-telluride blood agar or modified Tinsdale agar (TIN). Haemophilus influenzae is commonly isolated using chocolate agar, as it provides both hemin (X factor) and NAD (V factor)

The image to the right presents protozoan organisms in the cyst stage of development. Identify the protozoan cyst in Image D. 1) Chilomastix mesnili cyst 2) Giardia lamblia cyst 3) Acanthamoeba species cyst 4) Entamoeba coli cyst

Chilomastix mesnili cyst Chilomastix mesnili cyst is the correct answer because they are lemon or pear-shaped with a hyaline knob. They only have one nucleus and a curved fibril alongside the cytostome, often called the "shephard's crook." Giardia lamblia cyst is incorrect because they are often oval with longitudinal fibers. Four nuclei are typically present. Image C. Acanthamoeba species cyst in incorrect because cysts have a single, round nucleus along with a large karyosome. A double wall is usually visible. Image B. Entamoeba coli cyst is incorrect because the cysts have eight nuclei, an eccentric karyosome, and clumped, uneven chromatin. Image A.

Which of the following amoeba cyst measures around 25 µm? 1) Entamoeba hartmanni 2) Entamoeba histolytica 3) Entamoeba coli 4) Balantidium coli

Entamoeba coli Entamoeba coli is correct. Entamoeba coli cyst measure between 10 - 35 µm. Size is an important parasite-identifying feature. It is especially significant in distinguishing Entamoeba histolytica from Entamoeba hartmanni. The size ranges of these and other amoebae often overlap. Entamoeba hartmanni is incorrect because this amoeba measures less than 10 µm. Entamoeba histolytica is incorrect because this organism because between 10 - 20 µm. Balantidium coli is incorrect because this amoeba measures between 50 - 75 µm.

All of the following would be suitable for identification of bacterial antigens, EXCEPT? 1) Agglutination Reactions 2) Gram Stain Reactions 3) Immunofluorescence Methods 4) Precipitin Reactions

Gram Stain Reactions Gram stain reactions will not help to identify bacterial antigens since it cannot help bind to cell membrane antigens. However, agglutination reactions, immunofluorescence methods, and precipitin reactions can each help to measure and/or identify bacterial antigens.

Who should a technologist contact if an organism is isolated that cannot be ruled out as a potential agent of bioterrorism? 1) The local police department 2) The Federal Bureau of Investigation (FBI) 3) The Laboratory Response Network (LRN) reference laboratory 4) The Centers for Disease Control (CDC)

Laboratory Response Network (LRN) reference laboratory. In the event that a potential agent of bioterrorism is suspected, the appropriate internal staff should contact your LRN reference laboratory. The sentinel laboratory should NOT make the decision that a bioterrorism event has occurred, thus should not contact law enforcement or public health officials.

The Knott technique serves as a means of identifying: 1) Plasmodium vivax 2) Babesia microti 3) Loa loa 4) Strongyloides stercoralis

Loa loa Loa loa is the correct answer. The Knott technique is a procedure in which the red blood cells are lysed and the bodies of microfilariae are killed and straightened for better identification. The specimen is then centrifuged to concentrate it and the sediment is spread on a side and stained for review. Plasmodium vivaxis best identified using thick and thin blood smears to see the parasite in red blood cells. Babesia microti are best identified using thick and thin blood smears. Strongyloides stercoralis may be identified using standard techniques (direct and concentrated wet preparations and permanent stains) used for examining stool specimens.

Which of the following parasites is/are considered as atrial ameba(e)? 1) Naegleria fowleri 2) Trichomonas vaginalis 3) Chilomastix mesnili 4) Toxoplasma gondii

Naegleria fowleri The term atrial, as it applies to parasites, refers to those organisms that settle outside of the intestinal tract. Although all of these organisms fit this definition, only Naegleria fowleri is a free-living amoebae found in fresh water habitats. It produces a diffuse and fulminant meningoencephalitis. Trichomonas vaginalis s a flagellated protozoan and is one of the most sexually transmitted diseases. Chilomastix mesnili is a flagellate and it is considered to be a nonpathogen. Toxoplasma gondii is a tissue coccidia whose infections resemble mononucleosis-like symptoms .In pregnant women and in anyone with a compromised immune system the infection can have severe consequences.

A gram-negative rod isolated from a burn patient produced a bluish green pigment and the following test results: Characteristic fruity odorTriple sugar iron (TSI) agar: Alkaline/alkalineMotility: PositiveOxidase: PositiveOxidative/fermentation (OF) glucose: Oxidative utilization only What is the MOST probable genus of this isolate? 1) Acinetobacter 2) Alcaligenes 3) Moraxella 4) Pseudomonas

Pseudomonas The organism that most closely fits the clinical and laboratory picture is a Pseudomonas species. This microbe is associated with a bluish green pigment as well as a characteristic fruity odor. The biochemical tests also match the expected results for Pseudomonas.

Which of the following describes the flagella on the bacterium in in image B? 1) Monotrichous 2) Amphitrichous 3) Lophotrichous 4) Peritrichous

Peritrichous Image B shows peritrichous flagella, which means that the flagella occur all over the surface of the bacteria. Monotrichous means that there is one flagellum at one end of the bacterium, which is shown in image A. Amphitrichous means that there is one flagellum at both ends of the bacterium, which is show in image C. Lophotrichous is not shown in any image, but indicates that there are multiple flagella at one end of the bacterium.

If a Brucella species is suspected in blood culture, it should be held for a maximum of how many days on a commercial blood culture system? 1) 2-5 2) 5-7 3) 10-14 4) 14-21

10-14 If a Brucella species is suspected in a blood culture it should be held for a maximum of 10-14 days on a commercial blood culture system. Brucella spp. are highly infectious and present an infection risk to laboratory workers. It is suggested that laboratories should hold blood cultures for 10-14 days in systems like BacT/Alert or BACTEC, although most isolates can be detected within 5-7 days.

All of the following organisms stain with an acid-fast stain, or one of its variants, EXCEPT? 1) Mycobacterium spp. Cryptosporidium spp. 3) Actinomyces spp. 4)Nocardia spp.

Actinomyces spp. Mycobacterium spp., Cryptosporidium spp., and Nocardia spp. each stain with a form of acid-fast stain. The only choice listed that does not is Actinomyces spp..

The small, slowly growing gray-white colonies seen on the blood term-78agar plate were recovered from a blood culture of a 40-year-old farm worker who developed fever and chills leading to septicemia, following ingestion of contaminated milk. The Gram stain of this organism is seen in the lower image revealing small gram-negative coccobacilli in loose clusters. Key to the identification of this blood culture isolate is the rapid urease reaction. From these observations, select from the choices the presumptive identification of this isolate. 1) Cardiobacterium hominis 2) Eikenella corrodens 3) Brucella species 4) Campylobacter fetus

Brucella species Brucella species is the correct response. Although the appearance of small, slow growing colonies on blood agar and clusters of gram-negative coccobacilli on Gram stain are not specific, these observations might lead to the performance of a urea test. If the urease reaction is strongly positive within a short period of incubation, a presumptive identification of Brucella species can be made. Being a potential bioterrorism agent, such isolates should be sent immediately to a reference or public health laboratory for species confirmation. Undulant fever is endogenous in goats, cattle, and other animals, transferred to humans by direct contact or after ingestion of contaminated milk or other dairy products. Cardiobacterium hominis colonies are also small and slow growing on blood agar, but the Gram stain is specific with gram-variable staining, slender, elongated cells that often line up in rosette-like clusters. The urease reaction is negative. C. hominis is normal microbiota of the nose, mouth, and throat and usually causes endocarditis following oral infections or dental procedures. Eikenella corrodens colonies are more rapidly growing and are relatively large on both blood and chocolate agars. Distinctive is the pitting of the agar surrounding the colonies ("corrodens"), particularly on chocolate agar, with a light gray-white pigmentation of the adjacent agar. The urease reaction is weakly positive, often after incubation that is more prolonged. The most recognizable characteristic of the isolate is a distinctive bleach-like odor. E. corrodens is normal microbiota in the mouth and bowels. It is often isolated from human bite wounds and may also cause endocarditis. Campylobacter fetus colonies are more rapidly growing, and are relatively large with a smooth surface and light pigmentation on blood agar. Elongated, slender, curved and spiral bacilli are seen on gram stain. The urease reaction is negative.

Which of these parasites are hermaphroditic in their adult phase? 1) Clonorchis sinensis 2) Schistosoma haematobium 3) Wuchereria bancrofti 4) Trichuris trichiura

Clonorchis sinensis Organisms that are hermaphroditic are capable of self-fertilization. Of the organisms listed, only Clonorchis sinensis has this ability. Schistosoma haematobium (fluke), Wuchereria bancrofti (filariae), and Trichuris trichiura (nematode) have separate male and female adults for reproduction.

Which of the following Corynebacterium species is lipophilic? 1) Corynebacterium jeikeium 2) Corynebacterium amycolatum 3) Corynebacterium ulcerans 4) Corynebacterium striatum

Corynebacterium jeikeium The genus Corynebacterium is a large, diverse group of bacteria with approximately 50 clinically significant species. Corynebacteria can be divided into nonlipophilic and lipophilic species. Lipophilic corynebacteria are often considered to be fastidious and grow slowly on culture media, taking up to 48 hours of incubation before growth is detected. Growth is enhanced if lipids are added to the culture medium. The only lipophilic organism in the choices above is Corynebacterium jeikeium. The other three species are nonlipophilic.

All of the following are roles of a sentinel microbiology laboratory, EXCEPT? 1) Rule out pathogens. 2) Determine that a bioterrorist event has occurred. 3) Transfer suspicious specimens to reference laboratories. 4) Be familiar with likely agents involved in a bio crime.

Determine that a bioterrorist event has occurred. The role of a sentinel laboratory is to receive patient samples, rule out pathogens, and transfer suspicious specimens to reference laboratories. A reference laboratory confirms the presence of biological terror agents. The Federal Bureau of Investigation (FBI) has primary responsibility when a bioterrorism event occurs. In addition, there are an estimated 2,500 private and commercial laboratories serving in the sentinel capacity in the United States.

The colonies illustrated on the blood and chocolate agar plates represent a slow-growing bacterial species that is endogenous in the oral cavity and upper respiratory tract. It may be recovered from human bite skin infections incurred during fist fights. Note the pitting of the agar surrounding the colonies growing in the chocolate agar plate. The Alk/Alk non fermentative reaction seen in the Kligler Iron Agar tube to the left in the lower photograph indicates the non-fermentation of glucose, also indicated by the lack of carbohydrate assimilation in the individual carbohydrate reaction tubes to the right of the KIA tube. A reduction of nitrate is seen and decarboxylation of ornithine is also positive. Based on these observations, select from the multiple choices listed the correct identification of this bacterial isolate: 1) Brucella species. 2) Eikenella corrodens 3) Francisella tularensis 4) Haemophilus influenzae

Eikenella corrodens Eikenella corrodens is the correct response. The growth of small non-hemolytic colonies on blood agar with growth of larger pitting colonies growing on chocolate agar provides a presumptive identification of E. corrodens. Gram stains reveal slender, Gram negative bacilli with rounded ends. The lack of growth on MacConkey agar along with the asaccharolytic carbohydrate reactions as illustrated in the photograph to the right confirms the identification of E. corrodens. The reduction of nitrates and ornithine decarboxylase are positive reactions. Eikenella corrodens is normal flora in the upper respiratory tract and oral cavity, but may play a role in periodontal disease and root canal infections or as an agent of skin infections at the site of a bite incurred during a fist fight. Brucella species colonies are small and slow-growing on blood agar. Clusters of Gram negative bacilli on Gram stain are not specific. Distinctive is a strong positive urease reaction after a short period of incubation providing for a presumptive identification of Brucella species. Acid is produced oxidatively from several carbohydrates and oxidase and catalase reductions reactions are positive. Should be handled under a biological safety cabinet as it is the most common laboratory-acquired infection but also a cause of biological warfare. Francisella tularensis colonies are slow-growing, smooth, and pin-point in size. Tiny, pale staining Gram negative cocco-bacilli are observed in Gram stains. If the isolate is worked up in the laboratory, oxidase is weakly reactive and acid is produced from glucose and maltose. Depending the clinical condition from which the specimen was obtained, such as from ulcero glandular infections suggestive of tularemia, the isolate should be sent to a reference laboratory soon after tiny pinpoint colonies are observed on blood agar. F. tularensis is characterized a biological agent; the specimen should be processed under a biological safety cabinet and referred to a public health laboratory for identification. Haemophilus influenzae colonies may only be considered if entire, smooth, yellow-pigmented colonies are observed on chocolate agar. Growth does not occur on blood agars, most of which are Factor V deficient. Gram stains, particularly those prepared from the original specimen, reveal small, pale-staining Gram negative coccobacilli with a few filamentous forms intermixed. Positive indole and urease reactions are observed with some biotypes, and acid is produced from glucose, ribose and xylose.

Which substance acts as a mordant in the Gram stain process? 1) Acid alcohol 2) Crystal violet 3) Safranin O 4) Gram's iodine

Gram's iodine In the Gram stain, Gram's iodine is applied after the primary stain, Crystal violet (CV). Gram's iodine acts as a mordant by enhancing the binding of Crystal violet to the cell wall. Acid alcohol is used to remove the crystal violet-iodine complex from the cell wall. The thickness of the peptidoglycan layer for Gram-positive organisms helps resist the decolorization step of the staining process. Crystal violet is the primary stain of the Gram stain procedure. It stains the cells purple regardless of the thickness of the cell wall. Safranin O is the counterstain that is only visible if the decolorization step removes the crystal violet-iodine complex from the cell wall.

Which one of the following statements is true regarding the 2009 H1N1 virus? 1) Novel 2009 H1N1 virus is an influenza B virus. 2) Novel 2009 H1N1 virus only infects pigs and horses. 3) In 2009, the novel H1N1 virus spread during warmer months. 4) The Novel H1N1 virus spread in colder months.

In 2009, the novel H1N1 virus spread during warmer months. The novel H1N1 virus of 2009 spread in warmer weather months, distinguishing it from seasonal flu viruses that are known to peak in cold weather months. H1N1 is a recombination of human, avian, and swine influenza viruses which is how it spread so fast and during warmer months. It can infect several types of animals and is an Influenza A virus.

Amoebae stained with this substance may be readily distinguished, since it enhances nuclear and structural detail. 1) Saline 2) Iodine 3) Polyvinyl alcohol 4) Methylene blue

Iodine Iodine is correct. Iodine can be used alone or used in combination with formaldehyde. Iodine can be used to identify Iodamoeba butschlii cyst by being absorbed into the glycogen vacuole. The glycogen vacuole will stain dark brown in color. At times, the stained glycogen vacuole is all that is seen. Saline is incorrect. Although amoebae may be seen when examined using saline, nuclear and structural detail is not as evident, since saline is a clear substance and does not provide a color enhancement. Polyvinyl alcohol is incorrect. The polyvinyl alcohol fixative (PVA) is not a stain, but is used as a preservative for cysts and trophozoites. Methylene blue is incorrect because this permanent stain is not used tor staining amoebae.

The cytochrome oxidase-positive bacterial species that is DNAse negative, ornithine decarboxylase positive, and is associated with gastroenteritis in children after the ingestion of contaminated water is most likely: 1) Aeromonas hydrophila 2) Salmonella enterica 3) Plesiomonas shigelloides 4) Shigella sonnei

Plesiomonas shigelloides Plesiomonas shigelloides is an oxidase positive organism that is associated with gastrointestinal infections in children. This infection is typically acquired after the ingestion of contaminated water. It is also DNAse and ornithine decarboxylase negative. Aeromonas hydrophila is similar to Pleisiomonas as they are both oxidase positive, associated with contaminated water, and cause gastrointestinal infections in children. However, Aeromonas hydrophila produces DNAse and it does not hydrolyze ornithine, two key characteristics by which they are separated from Plesiomonas shigelloides. Salmonella enteritidis is similar to Pleisiomonas as it causes gastrointestinal infections and it is ornithine decarboxylase positive and DNAse negative. The key characteristic that differentiates it from Pleisiomonas is that it is oxidase negative. Shigella sonnei is similar to Pleisiomonas as it can cause gastroenteritis. It is also ornithine decarboxylase positive and DNAse negative; however, it differs from Pleisiomonas as it is cytochrome oxidase negative.

Which Pseudomonas species will have yellow colony pigmentation on blood agar? 1) Pseudomonas fluorescens 2) Pseudomonas aeruginosa 3) Pseudomonas mosselii 4) Pseudomonas fulva

Pseudomonas aeruginosa Pseudomonas fulva will show yellow pigmentation on blood agar. Pseudomonas fluorescens has no distinctive appearance on blood agar. Pseudomonas aeruginosa will have bluish green, red, or brown pigmentation observed on blood agar. Pseudomonas mosselii has no distinctive appearance on blood agar.

Baseline testing for latent Mycobacterium tuberculosis infection (LTBI) is recommended for all newly-hired healthcare workers. In which of these cases would a tuberculin skin test (TST) be repeated a second time, one to three weeks after the first test? 1) The first test is negative and the new employee has never been tested for LTBI before. 2) The first test is negative and the new employee has documentation showing he/she had a negative TST 6 months ago. 3) The first test is positive. 4) A positive Interferon Gamma Release Assay (IGRA) blood test.

The first test is negative and the new employee has never been tested for LTBI before. First test is negative and they haven't been tested previously. A negative TST is repeated 1 - 3 weeks after the first test if the individual being tested does not have a documented record of a negative test that occurred less than or equal to 12 months prior to new employment. A positive TST result should be followed up with additional testing to exclude TB disease such as chest X-ray and/or blood testing for TB disease to rule out LTBI (latent TB infection). IGRA is a blood test for determining exposure to the bacteria causing TB. A positive test is followed through similarly to a positive TST.

Nematodes are round and elongated and typically look like an earthworm. Nematodes can be found in decaying vegetation or moist soil. Of the following organisms listed, which one is considered a nematode? 1) Trichuris trichiura 2) Diphyllobothrium latum 3) Taenia solium 4) Hymenolepis diminuta

Trichuris trichiura Trichuris trichiura or whipworm is common in tropical or subtropical areas. It is the second most common nematode in the United States. Diphyllobothrium latum, Taenia solium, and Hymenolepis diminuta are all cestodes. Cestodes are also known as tape worms and are capable of self-fertilization.

Which of the following parasites has a life cycle that most closely resembles that of Plasmodium species? 1) Dracunculus medinensis 2) Blastocystis hominis 3) Babesia microti 4) Toxoplasma gondii

Babesia microti Like Plasmodium spp., Babesia spp. is a protozoan which infects red blood cells and produces a febrile illness. Babesia spp. also have sexual and asexual reproduction stages in the life cycle. Unlike the causative agents of malaria, which are spread by mosquitoes, Babesia organisms are transmitted by ticks. Dracunculus medinensis infection occurs when humans ingest copepods containing infective parasite larva. Once the larva is released in the intestine, it will penetrate the intestinal wall and migrate to a body cavity where the male and female parasites mature. A mature, gravid female will migrate through subcutaneous tissue to release liveborn larvae into water when the human comes into contact with the water. While the life cycle of Blastocystis hominis is not completely known, it is most likely that the infective cyst is ingested and exocysts in the large intestine where it will transform into a central vacuolar form. Toxoplasma gondii is an obligate intracellular parasite found in mammals. Humans will acquire infection either by ingesting or inhaling the oocyst or by ingesting bradyzoites in undercooked meat. The organism will invade cells in various organs, not red blood cells.

Group A beta-hemolytic streptococci are best characterized by which of the following: 1) Positive CAMP test 2) Optochin sensitivity 3) Bile esculin-hydrolysis 4) Bacitracin sensitivity

Bacitracin sensitivity Group A streptococci are best characterized by their susceptibility to bacitracin. Bacitracin disks are used as rapid screening tools to differentiate group A streptococci from other beta hemolytic streptococci. The CAMP test is used for the identification of group B streptococci. It detects the production of a diffusible, extracellular protein that enhances the hemolysis of sheep red blood cells by Staphylococcus aureus. Optochin sensitivity is used to differentiate Streptococcus pneumoniae (sensitive) from other alpha hemolytic streptococci (resistant). The hydrolysis of bile esculin is a characteristic used to differentiate enterococci from other streptococci.

In the formalin-ethyl acetate sedimentation procedure, four layers will occur. Out of the layers, where does the formalin layer occur? 1) Top layer 2) Bottom layer 3) Between the bottom layer and the debris/fat layer 4) Below the top layer and above the debris/fat layer

Between the bottom layer and the debris/fat layer Between the bottom layer and the debris/fat layer is the correct answer because formalin will have a lower density than the sediment and higher density than debris/fat. Top layer is incorrect because the top layer consists of ethyl acetate with the lowest density. The top layer consists of the "lightest" material. Bottom layer is incorrect because this is the sediment layer. Since parasites are "heavy" in this setting, they tend to settle in the bottom sediment layer along with other "heavy" fecal matter. Below the top layer and above the debris/fat layer is incorrect because there is nothing between these two layers. The formalin-ethyl acetate sedimentation procedure allows the recovery of protozoan trophozoites, cysts, helminth eggs, and larva. Formalin is used to preserve the parasites and the ethyl acetate is used to extract debris and fat from the stool while leaving the parasites in the sediment.

The following Clostridium difficile virulence factor is significant in the cause of antibiotic-associated pseudomembranous colitis: 1) Alpha toxin 2) Tetanospasmin 3) Neurotoxin 4) Cytotoxin A or B

Cytotoxin A or B Clostridium difficile is part of the human gastrointestinal microbiota. It becomes significant as a cause antibiotic-associated pseudomembranous colitis when the normal gastrointestinal flora is diminished, frequently due to the long-term or broad-spectrum use of antibiotics, along with the production of the virulence factors Cytotoxin A and/or B. The alpha exotoxin is a significant virulence factor for Clostridium perfringens. It causes the destruction of host cell membranes and attributes to myonecrosis (gas gangrene) and necrotizing fasciitis. Tetanospasmin, produced by Clostridium tetani, causes muscle spasms that are characteristic of the disease tetanus. Untreated cases result in death. A preformed neurotoxin, produced by Clostridium botulinum, may be found in non-acidic foods such as vegetables. The ingestion of this toxin may result in foodborne botulism that causes flaccid paralysis of the respiratory muscles and other muscles.

It is important to make a presumptive identification of the large, flat, spreading, gray-white, non-hemolytic colonies, with comma-like extensions growing on blood agar, as illustrated in the upper photograph. When mucoid colonies are observed, peak-like extensions may be observed when the colony is lifted with an inoculating loop (lower left image in the composite). Long, spore-bearing, Gram positive bacilli, often in straight and branching chains, are observed on Gram stain. Based on the results given, select the presumptive identification of this isolate. 1) Corynebacterium jeikeium 2) Bacillus anthracis 3) Lactobacillus species 4) Listeria monocytogenes

Bacillus anthracis Bacillus anthracis is the correct response. It is important to immediately recognize the large, flat, spreading, non-hemolytic gray-yellow colonies on blood agar with a ground-glass surface and irregular margins with comma-shaped extensions. The presumptive identification of Bacillus anthracis is supported by the peak like extension from the tip of an inoculating wire and the long chains of elongated, spore-bearing gram positive bacilli arranged in long branching chains. With these observations, the culture should be immediately referred to the local state laboratory to confirm this anthrax-producing agent of bio-terrorism. Corynebacterium jeikeium colonies are small, 0.5 - 1.0 mm, gray white, without peripheral extensions and are non-hemolytic. Microscopically are observed short Gram-positive bacilli arranged in V-forms or palisades, never in chains. Spores are not produced. Lactobacillus species colonies on sheep blood agar after 24 hours incubation are small, gray white, convex, and smooth without peripheral extensions. Light alpha hemolysis may be observed. Microscopic examination of Gram stained mounts reveal long, slender, non-spore forming positive bacilli also arranged in chains. Listeria monocytogenes colonies growing on blood agar are small, smooth, gray-white, and surrounded by narrow zones of soft beta hemolysis. Microscopically are observed small non-spore forming gram-positive bacilli with rounded ends lying singly in loose diphtheroidal-like clusters. The CAMP test is positive but with rectangular external rather than inverse zones of hemolysis.

Illustrated in the top photograph is a 4-day-old white to cream-colored colony with a yeast-like background, but with the surface having a delicate silk-like consistency. Such colonies may be recovered as a contaminant in laboratory cultures. The identification is illustrated in a cornmeal agar mount shown in the bottom photograph. From these observations, the MOST likely identification of this yeast-like isolate is: 1) Trichosporon beigelii 2) Aureobasidium pullulans 3) Geotrichum candidum 4) Phaeoannelomyces werneckii

Geotrichum candidum Geotrichum candidum has the microscopic appearance depicted in the bottom photograph (short septate hyphae, some of which may produce one of the corners a single germ tube from simulating a "hockey stick"). Although the colony is not specific, the silk-like aerial mycelium possibly suggest one of the yeast-like fungi that produces true hyphae. With some species the hyphae may be transformed into arthroconidia. Aureobasidium pullulans are characterized as "black yeasts" with dark pigmentation of colonies with a smooth, pasty surface. Dark-staining, non-budding arthroconidia in chains, with small single celled, non-encapsulated microconidia seen microscopically. Trichosporon beigelii colonies may also be yeast-like with a hair-like surface. True hyphae and budding arthroconidia are observed microscopically with the distinction that buds are from each of two corners forming what are called "rabbit ears", in contrast to the single "hockey stick" characteristic of Geotrichum. Phaeoannelomyces werneckiialso produces "black yeast" colonies. Elongated two celled yeast cells are produced in which the daughter cell is separated from the mother cell by a distinct deeply staining transverse bar called an "annelide", which can be microscopically observed.

Which of the following is the most likely identification of an acid fast bacilli recovered from an induced sputum with these culture findings? Slow growth at 37o C Niacin test = negative Pigmented photochromogenic Nitrate test = positive Catalase = positive 1) Mycobacterium kansasii 2) Mycobacterium tuberculosis 3) Mycobacterium simiae 4) Mycobacterium chelonei

Mycobacterium kansasii. Mycobacterium kansasii colonies are photochromagenic (form a pigment when exposed to light, but are non-pigmented in the dark); they are strongly catalase positive, strong nitrate reducers, and negative for niacin accumulation. M. kansasii produces a chronic lung infection that closely resembles pulmonary tuberculosis. Mycobacterium tuberculosis is positive for catalase production, able to reduce nitrate, but it is non-pigmented and positive for niacin accumulation, which does not fit with the information given above. Mycobacterium simiae is photochromogenic and catalase positive, but it does not reduce nitrate and it is positive for niacin accumulation. Mycobacterium chelonei is a rapidly growing mycobacterium that is also non-pigmented and does not reduce nitrate.

The accumulation of niacin, as shown by the yellow reaction on the test strip shown in the right tube is one of the chief characteristics to identify Mycobacterium tuberculosis. Another rarely encountered Mycobacterium species that also accumulates niacin is: 1) Mycobacterium gordonae 2) Mycobacterium simiae 3) Mycobacterium xenopi 4) Mycobacterium fortuitum

Mycobacterium simiae In practice, since M. simiae is an extremely rare isolate, any unknown Mycobacterium species that accumulates niacin can be identified as M. tuberculosis. However, M. tuberculosis is not the only species that accumulates niacin. Occasional strains of M. marinum, M.africanum, M. bovis, and M. chelonae may also accumulate niacin. M. simiae can be differentiated from M. tuberculosis by the nitrate and semi-quantitative catalase testing. M. tuberculosis is nitrate positive and semi-quantitative catalase negative. M. simiae is nitrate negative and semi-quantitative catalase positive. Mycobacterium gordonae is niacin negative. It is typically differentiated by it's orange colony color, opposed to the buff colored colonies of M. simiae and M. tuberculosis. It is also Tween hydrolysis positive and nitrate negative. Mycobacterium xenopi is niacin negative. It is a buff colored colony as well and is differentiated because it is nitrate negative, tween hydrolysis negative and semi-quantitative catalase negative. Mycobacterium fortuitum is niacin negative. It is a rapid growing species that is positive for the 3-day arylsulfatase test. It is also nitrate positive.

All of the following statements about peptide nucleic acid-fluorescence in situ hybridization (PNA-FISH) are true for Staphylococcus EXCEPT? 1) PNA probes are built with a pseudopeptide, rather than a sugar. 2) PNA-FISH assays provide differentiation between Methicillin Resistant Staphylococcus aureus (MRSA) and Methicillin Sensitive Staphylococcus aureus (MSSA). 3) PNA-FISH assays provide differentiation between Staphylococcus aureus and coagulase negative species. 4) PNA-FISH assays offer the possibility of earlier adjustment of antimicrobial therapy.

PNA-FISH assays provide differentiation between Methicillin Resistant Staphylococcus aureus (MRSA) and Methicillin Sensitive Staphylococcus aureus (MSSA). PNA-FISH assays do NOT differentiate between Methicillin Sensitive Staphylococcus aureus and Methicillin Resistant Staphylococcus aureus. PNA probes are built with a pseudopeptide, rather than a sugar is incorrect because this is true. The sugar phosphate backbone of DNA is replaced with polyamide backbone. PNA-FISH assays provide differentiation between Staphylococcus aureus and coagulase negative species is incorrect because this is a true statement. PNA-FISH assays offer the possibility of earlier adjustment of antimicrobial therapy is incorrect because differentiation between Staphylococcus aureus and a coagulase negative Staphylococcus species can be useful by discontinuing vancomycin treatment in scenarios where contamination with coagulase negative staphylococcus (rather than actual bloodstream infection) is indicated.

What is this form, measuring 45 µm and found in stool? 1) Balantidium coli cyst 2) Ascaris lumbricoides egg 3) Schistosoma mansoni egg 4) Hymenolepis diminuta egg

Ascaris lumbricoides egg This image shows a round egg with a distinct corticated outer shell, which is definitive for Ascaris lumbricoides. Balantidium coli cysts are round, but have a distinctive large, kidney-bean shaped macronucleus present in the cytoplasm. There can also be a visible micronucleus located adjacent to the macronucleus. Vacuoles can also be present. This image does not show a nucleus, so cysts and trophozoite forms can be ruled out. Schistosoma mansoni eggs can be distinguished by a large lateral spine present on the egg. They are also a much larger egg, ranging from 112 to 182 µm in size. Hymenolepis diminuta eggs do have a capsule around the embryo which can look similar to the Ascaris lumbricoides corticated shell, but Hymenolepis diminuta contains three pairs of hooks in the embryo that are distinguishable. Hymenolepis is also larger, typically 55 by 85 µm in size.

Illustrated in the top photomicrograph is a 4 day old colony growing on Sabouraud's dextrose agar with a smooth dark gray to black surface surrounded by dark gray and white peripheral bands. The colony morphology is not distinctive. The identification is made by observing the septate hyphae and conidia as microscopically observed in the lactophenol cotton blue stained mount obtained from the surface of the colony as illustrated in the bottom photomicrograph. From these observations, select the most probable fungi. 1) Exserohilum species 2) Curvularia species 3) Bipolaris species 4) Nigrospora species

Bipolaris species Bipolaris species is the correct answer. Distinctive for Bipolaris species is the production of elliptical to oval multi-celled macroconidia divided by transverse septa only. They are produced from the tips of curved septate conidiophores in a loose clustered, sympodial arrangement. The designation Bipolaris is derived from the property of producing germ tubes that arise from both end cells of the macroconidia as observed in saline mounts incubated at 25o C for 8 - 24 hours. Bipolaris is an important cause of fungal keratitis following eye injury in immunocompromised hosts. Curvularia species is incorrect. Macroconidia of Curvularia species are relatively large, distinctly with four or five cells that are separated by transverse septa, produced sympodially also in loose clusters from twisted conidiophores. The second cell from the terminal end of each macroconidium continues to grow after the others have matured, becoming enlarged to impart a curved shape that has been described as a "boomerang". Exserohilum species is incorrect. Macroconidia of Exserohilum species are similar to those of Bipolaris species, except that they are longer, have more cells/conidium, and characteristically have an extended, prominent protruding extension from the hilar cell. Nigrospora species is incorrect. Conidia of Nigrospora species are solitary, relatively large, subglobose, smooth, jet black, and borne from the tips of short inflated bulbous conidiophores.

The suspicious form pictured here in a red cell (lacking Schüffner dots) is responsible for which of the following conditions? 1) Benign tertian malaria 2) Black water fever 3) Filariasis 4) Chagas disease

Black water fever The organism pictured here is the ring form of Plasmodium falciparum, which is characterized by the presence of double rings that infect unsuspecting red blood cells. Furthermore, rings with 2 chromatin dots are suggestive of this organism. Plasmodium falciparum is the causative agent of black water fever. Plasmodium vivax is associated with benign tertian malaria and the red cells with have Schüffner dots. Blood and tissue filarial nematodes are roundworms that infect humans. When the arthropod vector feeds on a human blood meal, the infective larvae and injected into the blood stream where they will migrate to the lymphatic vessels. Wuchereria bancrofti is the most common identified species of filarial worms that infect humans and it causes bancroftian filariasis and elephantiasis. Trypanosoma cruzi causes Chagas disease and is found in the blood as trypomastigotes, which are about 20 µm long and it assumes a C or U shape in stained films.

The most common cause of acute meningitis associated with CSF shunts is: 1) Coagulase-negative staphylococci 2) Naegleria fowleri 3) Citrobacter koseri 4) Streptococcus pneumoniae

Coagulase-negative staphylococci Coagulase-negative staphylococci cause approximately 50% of acute meningitis cases associated with CSF shunt infections. Other skin flora such as species of Corynebacterium and Cutibacterium (Propionibacterium) acnes also are leading causes of CSF shunt infections. Naegleria fowleri causes primary amoebic meningoencephalitis, Citrobacter koseri is a cause of neonatal meningoencephalitis, and Streptococcus pneumoniae remains an important cause of meningitis in adults.

Loeffler's medium is useful in making the presumptive identification of isolates suspicious for Corynebacterium diphtheriae recovered from oropharyngeal cultures because it: 1) Inhibits the growth of contaminating bacteria 2) Detects the production of H2S 3) Enhances granule formation as seen in methylene blue stains 4) Screens out the toxin producing strains

Enhances granule formation as seen in methylene blue stains Although the identification of Corynebacterium diphtheriae in upper respiratory specimens cannot be made by the study of direct smears, the accentuated granule formation seen in methylene blue stains of isolates grown on this medium is a helpful property in determining which strains should be further studied, either by subculture to tellurite medium or for toxicity studies. Loeffler's medium is a rich, non-inhibitory culture medium on which many bacterial species readily grow; however, bacterial cells suspicious for Corynebacterium diphtheriae can be readily identified because of the metachromatic granules (Babes-Ernst granules). Corynebacterium diphtheriae does not produce H2S gas and there are no detectors in the medium even if the gas were produced. The medium does not differentiate toxin-producing from non-toxin-producing strains.

Not all strains of Entamoeba histolytica appear to be pathogenic, and recent molecular studies indicated that there was a non-pathogenic variant of Entamoeba histolytica. What is this non-pathogenic variant? 1) Entamoeba hartmanni 2) Entamoeba moshkovkii 3) Entamoeba dispar 4) Entamoeba coli

Entamoeba dispar Several DNA sequencing assays are available to separate the non-pathogenic variant, Entamoeba dispar, from Entamoeba histolytica. It is important to know that these assays are available in cases where the recovery of an isolate identified as Entamoeba histolytica by conventional methods may not have a clinical correlation. Entamoeba hartmanni is similar to E. histolytica but does not possess the exclusive genetic composition representing pathogenicity. Entamoeba moshkovkii is an infectious agent to humans and is morphologically identical to E. hystolytica. It is identified as a GI pathogen. Entamoeba coli is not pathogenic, but it is easily distinguished from E. histolytica. As the trophozoites and cysts of Entamoeba coli are easy to identify when seen in stool specimens, antigen testing has not been necessary.

A fungus culture from a toe-nail scraping demonstrates large, club-shaped macroconidia with no microconidia. The most likely identification is: 1) Trichophyton mentagrophytes 2) Microsporum canis 3) Epidermophyton floccosum 4) Trichophyton gypseu

Epidermophyton floccosum Epidermophytonfloccosum is a common cause of tinea pedis (athlete's foot). It characteristically numerous, smooth, multiseptate, thin-walled macroconidia that are club-shaped with no microconidia. Trichophyton mentagrophytes are common causes of feet and nail fungal infections. The mold produces a cherry-red reverse color. Microscopically, it produces numerous microconidia in grape-like clusters along the hyphae and a few club-shaped, smooth-walled macroconidia. Microsporum canis produces hair and skin fungal infections. The mold colony typically has a lemon-yellow or yellow-orange apron or reverse. Microscopically, it produces thick-walled, spindle-shaped, rough-walled, multicellular macroconidia with rare microconidia. Trichophyton gypseum produce hair, skin, or nail infections. The mold colony has a cinnamon-colored, powdery surface with a tan reverse. Microscopically, it produces thick-walled, rough, spindle-shaped with round ends, multiseptate macroconidia, and rare microconidia.

Illustrated in the image shown is a 20 µm amoebic trophozoite that was observed in a random stool specimen from a patient who had no symptoms of a parasitic infection. Several molecular based studies have found that in the analysis of stable DNA polymorphisms, several differences in DNA zymodeme patterns were found to encode for genetically different strains of Entamoeba histolytica. As some of these strains were found to be non-pathogenic, it has been recommended that the laboratory report be altered to draw attention to this possibility. Select from the choices below the report that should be issued. 1) Entamoeba histolytica/dispar 2) Entamoeba histolytica/polecki 3) Entamoeba hartmanni 4) Entamoeba coli

Entamoeba histolytica/dispar Entamoeba histolytica/dispar is the correct response when isolated trophozoites as illustrated in the photomicrograph are observed. Issuing this report indicates that therapy may not be required depending on the clinical history and physical findings. If the differentiation between Entamoeba histolytica and Entamoeba dispar is required, antigen detection reagent kits are currently available for the performance of on-site assays, or for transport of fresh stool specimens to a reference laboratory. Entamoeba dispar does not invade tissue and red blood cells are rarely ingested. Entamoeba histolytica/polecki is an incorrect response. This species, endogenous in pigs, is rarely encountered in human stool specimens. Entamoeba hartmanni is an incorrect response. Entamoeba hartmanni trophozoites are similar in morphology to those of Entamoeba histolytica, but are small usually ranging <10 µm. They are rarely seen in stool specimens and could be identified by searching for the small cysts usually with no more than 2 nuclei and a chromatoidal bar with rounded ends. Entamoeba coli is an incorrect response. Entamoeba coli trophozoites are distinctive for the nucleus that has a large, eccentrically placed karyosome, a blotchy distribution of chromatin around the external ring, and cytoplasm that is coarse and junky rather than smooth and evenly distributed. This species is not pathogenic.

All of the following statements concerning biochemical reactions are true, EXCEPT: 1) Enterococcus faecalis is positive for esculin hydrolysis, as depicted in tube C. 2) Bordetella bronchiseptica gives a strong positive urease reaction as depicted in tube B. 3) Escherichia coli is positive for utilization of sodium citrate, as depicted in tube A. 4) Morganella morganii is positive for phenylalanine deaminase, as depicted in tube D.

Escherichia coli is positive for utilization of sodium citrate, as depicted in tube A. The correct answer is Escherichia coli is positive for utilization of sodium citrate, as depicted in tube A. Escherichia coli is negative for utilization of sodium citrate, but in the image, tube A does display a positive reaction. All of the tubes in the image display positive reactions for their respective test. The organisms given are those most commonly associated with the positive result observed. Tube A: sodium citrate utilization positive Tube B: urea hydrolysis positive Tube C: esculin hydrolysis positive Tube D: phenylalanine deaminase positive

The common name of which parasite is the sheep liver fluke? 1) Fasciolopsis buski 2) Fasciola hepatica 3) Clonorchis sinensis 4) Paragonimus westermani

Fasciola hepatica Fasciola hepatica is commonly known as the sheep liver fluke. When the organism infects sheep, it causes a disease known as liver rot. Humans ingest metacercaria on raw water vegetation. Once ingested the larvae migrate through the intestinal wall and peritoneal cavity and travel to the liver. In general terms, the common names of these particular parasites are derived from the area of the human body where they tend to inhabit. Knowledge of the parasite life cycles is helpful in making this determination. Fasciolopsis buski is commonly referred to as the giant intestinal fluke, Clonorchis sinensis is the Chinese liver fluke and Paragonimus westermani is known as the lung fluke.

In tissues infected with Histoplasma capsulatum, which of the following is usually true? 1) Hyphae usually invade blood vessels 2) Encapsulated yeast cells are typical 3) Conidiophores with metulae and phialides are typical 4) Fungus is usually intracellular

Fungus is usually intracellular Fungus is usually intracellular is correct because Histoplasma capsulatum is found within mononuclear cells as small, round, to oval yeast cells. Histoplasma capsulatum also invades the reticuloendothelial system, which is the primary site of dissemination through the lymph nodes, liver, spleen, and the bone marrow. Hyphae usually invade blood vessels is incorrect because Histoplasma capsulatum does not invade the blood vessels. This is an example of a mucomycosis infection caused by the Mucorales group (Rhizopus, Mucor, and Rhizomucor, etc.). Encapsulated yeast cells are typical is incorrect because Cryptococcus neoformans is considered an encapsulated yeast cell. The India Ink stain is a rapid method to determine if Cryptococcus neoformans is present in cerebrospinal fluid. Conidiophores with metulae and phialides are typical is incorrect as this is a description of Aspergillus species.

Which of the following species of bacteria is positive for rapid urease production, as demonstrated by a color change in urea agar from yellow to pink within two hours? Image courtesy of the CDC 1) Salmonella enterica 2) Escherichia coli 3) Helicobacter pylori 4) Campylobacter jejuni

Helicobacter pylori Rapid urease testing may be useful in the presumptive identification of Helicobacter pylori from gastric biopsy specimens. Positive results are indicated by a color change from yellow to pink, as the ammonia produced from urea hydrolysis creates alkalinization using phenol red as the pH indicator. Salmonella enterica, Escherichia coli, and Campylobacter jejuni are also Gram-negative rods implicated in enteric infections, but are urease negative. Stool specimens are routinely screened for Salmonella, Escherichia, and Campylobacter.

The process of freeze-drying of bacteria is called: 1) Agglutination 2) Inhibition 3) Lyophilization 4) Precipitation

Lyophilization The process of quick-freeze drying of bacteria is called lyophilization. Freeze-drying or lyophilizing bacteria is a successful method for long-term preservation. Freeze drying bacteria requires multiple steps, including culturing the microbes, suspending them in a lyophilization medium/buffer, subjecting them to the freeze-drying process, and storing them appropriately. Agglutination is the aggregation or clumping of particles; inhibition prevents visible results or growth, such as when doing a MIC; precipitation is the immunologic reaction between soluble antigens and antibodies forming visible macromolecular complexes.

Performing quantitative cultures on bronchoalveolar lavage and protected brush specimens are better than performing routine semi-quantitative cultures because in quantitative cultures there should be which of the following? 1) Low numbers of contaminating normal respiratory flora and colonizing organisms and higher numbers of potential pathogens. 2) Low numbers of contaminating normal respiratory flora and colonizing organisms and low numbers of potential pathogens. 3) High numbers of contaminating normal respiratory flora and colonizing organisms and higher numbers of potential pathogens. 4) High numbers of contaminating normal respiratory flora and colonizing organisms and lower numbers of potential pathogens.

Low numbers of contaminating normal respiratory flora and colonizing organisms and higher numbers of potential pathogens. Low numbers of contaminating normal respiratory flora and colonizing organisms and higher numbers of potential pathogens is correct because when obtaining these specimens, the respiratory tract is bypassed by using a bronchoscope, which will allow recovery of a larger number of pathogens and prevent contamination from the upper respiratory tract. In a BAL procedure, a large amount of saline is infused into a lung segment through a bronchoscope, which will allow the collection of cells and protein from the pulmonary interstitium and alveolar spaces from the area in question. A protected brush is a small brush that is inserted through a bronchoscope to the area of interest. Both processes allows for the bypass of contamination from upper respiratory flora. Low numbers of contaminating normal respiratory flora and colonizing organisms and low numbers of potential pathogens is incorrect because the goal is to recover a higher number of pathogens to diagnosis a true bacterial infection instead of low numbers of pathogens. High numbers of contaminating normal respiratory flora and colonizing organisms and higher numbers of potential pathogens is incorrect because the goal of using these procedures is to prevent contamination of the specimen with normal respiratory flora and colonizing organisms. High numbers of contaminating normal respiratory flora and colonizing organisms and lower numbers of potential pathogens is incorrect because these procedures are used to reduce contamination of specimens with normal respiratory flora and colonizing organisms while collecting a higher yield of potential pathogens.

Carbapenemases are produced by various organisms such as Enterobacteriaceae, Pseudomonas aeruginosa, and Acinetobacter species. Detecting the presence of carbapenemase activity in these organisms is important to prevent spreading of the resistant organisms. Which of the following methods is used to detect the presence of carbapenemase producing organisms? 1) D test 2) Modified Hodge Test 3) ESBL Test 4) Beta-lactamase test

Modified Hodge Test Modified Hodge Test is the correct answer because the presence of a carbapenemase can be detected by this method. This method is also referred to as the clover-leaf test. This test uses a carbapenem-susceptible Escherichia coli that is plated to a Mueller-Hinton agar and then carbapenem disks are placed onto the plate. The test organisms are inoculated in a straight line from the disks and growth of the Escherichia coli around the test organism at the intersection of the streak and the zone of inhibition indicates the presence of a carbapenemase. D test is incorrect. The D test is used to detect inducible clindamycin resistance by plating the test organism on a Mueller-Hinton plate and then placing erythromycin and clindamycin disks. If a D zone of inhibition is present after a 24 hour incubation at 37° C, the organism has an inducible resistance to clindamycin, due to the erm gene. ESBL Test is incorrect. This test detects organisms that produce extended spectrum beta-lactamases to the cephalosporin class of antibiotics. To detect an ESBL producing organism, antibiotic disks cefotaxime and cefotaxime-clavulanate and ceftaxidime and ceftaxidime-clavulanate are placed on a Mueller-Hinton agar plated with the test organism. If there is a zone difference between the antibiotic with and without the clavulanate, then the presence of an ESBL is indicated. Beta-lactamase test is incorrect. This test is a rapid test to detect penicillin resistance. Beta-lactamase breaks down the beta-lactam ring found in penicillin.

Which of the following choices represents the characteristic indicator, fermentable sugar(s), and bacteriostatic agent(s) that are present in MacConkey agar respectively? 1) Phenol red/lactose, sucrose, and xylose/sodium desoxycholate and bile salts 2) Eosin Y and methylene blue/lactose and sucrose/Eosin Y and methylene blue 3) Bromothymol blue/lactose, salicin, and sucrose/bile salts 4) Neutral red/lactose/bile salts and crystal violet

Neutral red/lactose/bile salts and crystal violet MacConkey agar contains neutral red, which is a dye that is absorbed by the colonies during fermentation. Lactose is the only sugar present in the medium, and lactose fermenters appear pink to red, and non-fermenters appear clear or colorless. The inhibitory agents are bile salts and crystal violet, Xylose-Lysine-Desoxycholate (XLD) agar uses phenol red as the pH indicator for fermentation. Lactose, sucrose, and xylose are present in the medium, and fermenters appear yellow, whereas non-fermenters appear clear or colorless. The medium also contains desoxycholate, which inhibits Gram-positive organisms. There are also bile salts in lower concentrations than other enteric media (such as Salmonella-Shigella or Hektoen Enteric) for better recovery of Shigella species. Eosin-Methylene Blue (EMB) agar uses Eosin Y and methylene blue to both allow the color change to occur while the fermentation of the sugar takes place, dropping the pH, and as the bacteriostatic agent by inhibiting the growth of Gram-positive bacteria. In addition, lactose and sucrose are incorporated into the medium, and lactose and sucrose fermenters appear blue/black, and non-fermenters appear clear and colorless. Hektoen Enteric (HE) agar uses bromothymol blue as the pH indicator for color change during fermentation. Lactose, salicin, and sucrose are all present in the medium, and fermenting colonies appear orange to salmon pink. Non-fermenting colonies appear green to blue-green. The inhibitory agent is bile salts at a concentration that inhibits Gram-positive organisms and many Gram-negative organisms that are normal fecal flora.

This parasite assumes a characteristic banana shape and grows inside red blood cells until it eventually destroys the cells. What is the MOST likely identification and stage? 1) Plasmodium falciparum gametocytes 2) Plasmodium ovale gametocytes 3) Plasmodium malariae gametocytes 4) Plasmodium vivax gametocytes

Plasmodium falciparum gametocytes term-18 The gametocytes of Plasmodium falciparum are the only malarial organisms that assume a characteristic banana shape. This shape is diagnostic for P.falciparum. These shapes are generally only present in the early stages of infection and disappear within 10 days of fever onset. Due to the risk of central nervous system involvement specifically with these species, it is important to rule it in or out as quickly as possible. Plasmodium ovale gametocytes are round, which differentiates them from P.falciparum, but not the other Plasmodium sp. Plasmodium malariae gametocytes are round and contained within a normal sized red blood cell. Plasmodium vivax gametocytes are large and circular. They continue to grow and occupy more than half of the cytoplasm.

Which malarial parasite is most likely present in a patient with a stained blood smear that reveals giant platelets, many reticulocytes containing Schuffner's dots, large ring forms, and schizonts with 12-24 merozoites? 1) Plasmodium falciparum 2) Plasmodium malariae 3) Plasmodium ovale 4) Plasmodium vivax

Plasmodium vivax Plasmodium vivax characteristically causes Schuffner's dots, ring forms in all stages of development, schizonts with 12-24 merozoites. Plasmodium falciparum does not cause Schuffner's dots to appear and schizonts are rarely observed in peripheral blood smears. Plasmodium malariae does not cause Schuffner's dots to appear and schizonts only contain between 6-12 merozoites. Plasmodium ovale exhibits James' stippling, which is indistinguishable for Schuffner's dots, and only produces schizonts with 6-12 merozoites.

A bacterium was isolated as the cause of a urinary tract infection. The organism showed no lactose fermentation on MacConkey agar (MAC). On triple sugar iron (TSI) media, the organism produced an A/A with H2S. What organism is most likely observed? 1) Enterobacter aerogenes 2) Proteus mirabilis 3) Proteus vulgaris 4) Edwardsiella tarda

Proteus vulgaris From the information given, the organism in question does not ferment lactose, but does ferment sucrose and glucose as determined from the MAC agar and TSI. The organism is also H2S positive. From the list of organisms, only Proteus vulgaris produces these reactions. Enterobacter aerogenes ferments lactose, sucrose, and glucose, but does not produce H2S. Proteus mirabilis is negative for lactose and sucrose fermentation, positive for glucose fermentation, and is H2S positive. Edwardsiella tarda is negative for lactose and sucrose fermentation, positive for glucose fermentation, and is H2S positive.

The disappearance of HBsAg and HBeAg, the persistence of Total anti-HBc, the appearance of anti-HBs, and often the presence of anti-HBe indicates what type of Hepatitis infection? 1) Acute HBV infection 2) Chronic HBV infection 3) Recovery phase of HBV infection 4) Acute HCV infection

Recovery phase of HBV infection The recovery phase of Hepatitis B viral infections show the actual viral antigens disappearing while the body produces an antibody to the virus's core, surface, and e antigens. In acute HBV infections, the HBsAg would be present along with HBeAg. In chronic HBV infections, the HBsAg, HBeAg, and Total Anti-HBc will be positive. Acute HCV infection is incorrect as the question is referring to Hepatitis B antigen and antibodies.

The ideal stool sample for parasitic examination is one that is freshly collected and submitted to the laboratory at: 1) Refrigerator temperature (2° C) 2) Incubation temperature (37° C) 3) A temperature of -70° C 4) Room temperature (approximately 22° C)

Room temperature (approximately 22° C) Room temperature is the ideal condition for the submission of stools for parasite examination. Because trophozoites disintegrate rapidly, liquid stool specimens should be examined within 30 minutes after collection and semisolid stools within 60 minutes of collection. If the specimen cannot be examined within these time frames, a preservative should be used. Specimens incubated, refrigerated, or frozen are more likely to produce unreliable results. Such extreme temperatures are known to destroy numerous parasites. Stool specimens should never be frozen and thawed or placed in an incubator because the parasitic forms may deteriorate rapidly.

Staphylococcus species recovered from a blood culture was found to produce acid from sucrose and maltose and showed alkaline phosphatase activity. The Staphylococcus was also coagulase negative. The same organism was also found in a culture from the central line tip. The most likely identification is: 1) Staphylococcus saprophyticus 2) Staphylococcus schleiferi 3) Staphylococcus epidermidis 4) Staphylococcus aureus

Staphylococcus epidermidis Staphylococcus epidermidis is a normal flora organism on the skin, but a common cause of infections from medical devices. The organism can produce biofilms on catheters and other medical devices. The biofilm is a complex interaction between the device, host, and bacteria. A large number of organisms may aggregate in the biofilm and then be shed into the bloodstream or other nearby sites depending on the biofilm placement. Sometimes in blood cultures, Staphylococcus epidermidis can be seen as a contaminant from the skin during collection. However, by the organism presence in both the blood and line cultures, this can be determined as an infection from the central line. S. epidermidis also produces acid from sucrose and maltose and has alkaline phosphatase activity, important characteristics included in most algorithms and identification systems. They are also coagulase negative. Staphylococcus saprophyticus is coagulase negative, and does produce acid from sucrose and maltose, but is alkaline phosphatase negative. These organisms are laso typically found in the urinary system as a cause of urinary tract infections, specifically in women, and rarely found on the skin or mucous membranes. Staphylococcus schleiferi is also coagulase negative Staphylococcus but it does not produce acid from sucrose and maltose. It is alkaline phosphatase positive. These organisms are also typically considered contaminants or causes of opportunistic infections. Staphylococcus aureus is a coagulase positive Staphylococcus and would be eliminated based on that result alone. S. aureus however, is alkaline phosphatase positive as well as produces acid from sucrose and maltose.

The federal government has categorized critical biological agents into three groups: A, B, and C. Why are the agents in category A classified as the highest-priority? 1) These organisms cause moderate morbidity and low mortality. 2) These organisms could be engineered for mass dissemination at some future date. 3) These organisms can be easily disseminated or transmitted person-to-person. 4) The organisms are not easily disseminated and do not cause high morbidity rates.

These organisms can be easily disseminated or transmitted person-to-person. The federal government has categorized critical biological agents into three groups: A, B, and C. Those agents in category A are highest-priority because they can be easily disseminated or transmitted person-to-person. Category B agents cause moderate morbidity and low mortality. Category C agents could be engineered for mass dissemination in the future because of availability, ease of production and dissemination, and the potential for high morbidity and mortality.

This suspicious form, found in stool, which measures 15 µm by 10 µm, is responsible for which of the following diseases? 1) Malaria 2) Traveler's diarrhea 3) None; considered as a non-pathogen 4) Amebiasis

Traveler's diarrhea. The organisms pictured here are the cysts of Giardia lamblia. These intestinal organisms are known to produce "traveler's diarrhea," named so since individuals contract this condition when they travel afar, especially to areas with poor sanitation. It is the most commonly reported protozoan in the United States. Malaria is caused by Plasmodium species and are recovered in blood. Amebiasis is caused by amoebas, such as Entamoeba histolytica, which has nuclear and structural features not in common with the G. lamblia.

If you receive a transudate fluid for a Gram stain and bacterial culture, what is the ideal specimen preparation method for the bacterial culture? 1) Use centrifugation to concentrate the sample; then using the pellet to inoculate the media. 2) Use centrifugation to concentrate the sample; then using the supernatant to inoculate the media. 3) Dip a swab in the original sample and use this swab to inoculate the media. 4) Use a sterile pipette to draw up the original sample and use this fluid to inoculate the media.

Use centrifugation to concentrate the sample; then using the pellet to inoculate the media. The correct answer is use centrifugation to concentrate the sample; then using the pellet to inoculate the media. "Thin" specimens, such as cerebrospinal fluid and transudate fluid, should be centrifuged to concentrate the sample. Using the pellet of the concentrated sample, inoculate the media by placing a single drop on each plate. Using the supernatant would not be useful in detecting bacterial pathogens that may be present in the sample, as all of the bacteria would likely be found in the pellet. Using a swab dipped in the original sample to inoculate the media would not be recommended, as swabs absorb more material than they are able to transfer to the media. Using a sterile pipette to draw up the original sample and inoculate the media would be acceptable, but not the ideal method for recovering bacteria present in the sample.

Pseudomonas aeruginosa was isolated from a sputum specimen. Susceptibility testing was performed and the microbiologist determined that further evaluation needed to take place before reporting out the susceptibilities. Which of the antibiotic results would indicate that further evaluation needed to be taken? 1) Amikacin resistant, Gentamicin or tobramycin sensitive, Carbapenem resistant 2) Amikacin sensitive, Gentamicin or tobramycin sensitive, Carbapenem resistant 3) Amikacin resistant, Gentamicin or tobramycin sensitive, Carbapenem sensitive 4) Amikacin sensitive, Gentamicin or tobramycin sensitive, Carbapenem sensitive

Amikacin resistant, Gentamicin or tobramycin sensitive, Carbapenem resistant Amikacin resistant, Gentamicin or tobramycin sensitive, Carbapenem resistant is the correct answer. Pseudomonas aeruginosa can develop resistance to antibiotics; however, when the following combination of resistance occurs, the results must be verified. Amikacin and carbapenem antibiotics, such as imipenem and meropenem, are used as a treatment option for resistant Pseudomonas aeruginosa isolates. Therapy for Pseudomonas aeruginosa involves the use of a beta-lactam drug with antipseudomonal activity and an aminoglycoside. Amikacin sensitive, Gentamicin or tobramycin sensitive, Carbapenem resistant is incorrect because resistance can occur with a carbapenem drug; however, when both amikacin and carbapenem drugs are resistant, a hard stop must be taken to verify accurate susceptibility results. Amikacin resistant, Gentamicin or tobramycin sensitive, Carbapenem sensitive is incorrect because resistance can occur with amikacin; however, when both amikacin and carbapenem drugs are resistant, a hard stop must be taken to verify accurate susceptibility results. Amikacin sensitive, Gentamicin or tobramycin sensitive, Carbapenem sensitive is incorrect because this susceptibility pattern does not require further evaluation as the organism can be treated with all drugs listed.

Which of the following is the correct classification for Image C? 1) Ciliate 2) Hemoflagellate 3) Flagellate 4) Ameoba

Ciliate Ciliate is the correct answer because this image represents a Balantidium coli trophozoite. The anterior end of a B. coli trophozoite is pointed and has a cytostome or mouth opening. Hemoflagellate is incorrect (Image B) because this image represents a Trypanosoma species. Flagellate is incorrect (Image A) because this image represents a Trichomonas species trophozoite. Amoeba is incorrect (Image D) because this image represents Iodamoeba butschlii cyst with a glycogen vacuole and a large karyosome.

From the choices listed below, select the biochemical reaction that excludes a bacterial isolate from the family Enterobacteriaceae. 1) Positive nitrate reduction 2) Positive cytochrome oxidase 3) Alk/Alk reaction on Kligler's Iron Agar 4) H2S Production

Alk/Alk reaction on Kligler's Iron Agar Alk/Alk reaction on Kligler Iron Agar is the correct response. The lack of acid production from either glucose or lactose is indicated by the non-pigmented Alk/Alk reaction on Kligler Iron Agar is sufficient to exclude a bacterial isolate from the Enterobacteriaceae. Alk/Alk means no carbohydrate fermentation. This is characteristic of nonfermentative bacteria such as Pseudomonas aeruginosa. The nitrate reduction test is used to determine the ability of an organism to reduce nitrate to nitrite. All members of the Enterobacteriaceae family reduce nitrate, but some members further metabolize nitrite to other components. The oxidase test determines the presence of cytochrome oxidase activity in microorganisms for the identification of oxidase-negative Enterobacteriaceae, differentiating them from other gram-negative bacilli. Some Enterobacteriaceae such as Salmonella spp. produce H2S. Members of Enterobacteriaceae generally lack cytochrome oxidase, although there are exceptions (e.g. Plesiomonas shigelloides), therefore, this test, as well as nitrate reduction and H2S production, cannot exclude a bacterial isolate from this family.

The pink-red, yeast-like colonies shown in the photograph together with the production of budding yeast cells singly and in loose clusters in the absence of hyphal formation is characteristic of which of the yeast listed below? (In the past, recovery in laboratory culture this organism was considered a contaminant; however, more recently it has been listed as an "emerging" agent of infection and cannot be considered as insignificant.) 1) Aureobasidium pullulans 2) Rhodotorula mucilaginosa 3) Trichosporon beigelii 4) Geotrichum candidum

Rhodotorula mucilaginosa Rhodotorula mucilaginosa is the correct response. In most mycology laboratory practices, the pink-red pigmentation of the yeast colonies would provide a visual presumptive identification. Most commonly the demonstration of a rapid positive spot urease reaction would be used to confirm the identification rather than setting up the streaking of cornmeal agar. Microscopically, small oval yeast cells are observed in mounts, some producing an elongated bud. Catheter-related infections, particularly peritonitis in patients receiving peritoneal dialysis, is one of the more common opportunistic infections caused by this yeast. Aureobasidium pullulans is incorrect. This species is characterized as "black yeasts". Microscopically, dark-staining, non-budding arthroconidia in chains, with small single celled, non-encapsulated microconidia seen in the background are produced. Trichosporon beigelii is incorrect. Colonies are gray-white with a hair-like mycelium and begins growth as a yeast. When mature, true hyphae and arthroconidia are formed that excludes this fungus from a yeast. Microscopically, the production of blastoconidia from both corners of arthroconidia simulating "rabbit ears" is the distinguishing identifying feature. Geotrichum candidum is incorrect. Colonies are gray white with a silk-like aerial mycelium and upon microscopic examination, continuous chains of arthroconidia are seen.

The fastidious gram negative bacillus, often producing pitting of the agar around the colonies growing on blood agar, as illustrated in this photograph, producing a bleach-like odor is most commonly associated with one or more of the following conditions: 1) "Clenched fist" infections of the hand 2) Juvenile periodontitis 3) Pneumonia 4) Dog bite cellulitis

"Clenched fist" infections of the hand. Eikenella corrodens is part of the normal flora of the upper respiratory tract of humans. Infections commonly occur when human saliva of one individual is introduced into the subcutaneous tissue of another, either directly from human bites or other forms of trauma such as occurs in bare knuckle fist fights ("clenched fist" infections). Intravenous drug users, who habitually lick the needle before insertion ("skin popping"), also have a high incidence of cellulitis from E. corrodens. Juvenile periodontitis is more commonly caused by another member of the HACEK group of fastidious gram negative bacilli, Actinobacillus actinomycetemcomefans; or, by Capnocytophaga species. Eikenella corrodens is not known to cause pneumonia E. corrodens does not reside in the oropharynx of dogs; therefore, is not related to infections caused by dog bites.

A 40-year-old man with a prosthetic valve developed intermittent bouts of low grade fever. A changing cardiac murmur suggested the possibility of endocarditis; however, repeat blood cultures over a two week period were all negative. Bacteria resembling those seen in the lower image were finally seen in Gram stain of a positive bottle. Colonies were recovered when broth was subcultured to a blood agar plate with a staph streak, as shown in the upper image. What is the most likely bacterial species? 1) Streptococcus mitis group 2) Enterococcus faecalis 3) Abiotrophia defectiva 4) Streptococcus bovis

Abiotrophia defectiva The images illustrate an organism that requires growth factors provided by the Staphylococcus shown in the streak preparation. Slow-growing Streptococcus-like species variously known as "satelliting," "nutritionally dependent," or "thiol requiring" need thiol compounds, such as cysteine or the active form of vitamin B6 (pyridoxal), added to culture media to promote growth. These compounds are synthesized by Staphylococcus species and other bacterial species, so that the nutritionally dependent Abiotrophia defectiva may appear as satellite colonies. Although the other bacterial species listed may appear as gram-positive cocci in chains and may cause endocarditis, none are thiol-dependent and would appear as a more diffuse growth on the blood agar surface, not confined to the area adjacent to the Staphylococcus streak shown in the upper image.

What is the first test always performed on organisms growing on solid or liquid mycobacterial media? 1) Rate of growth 2) Pigment production 3) Catalase 4) Acid-fast staining

Acid-fast staining Regardless of the identification methods used, the first test always performed on organisms growing on solid or liquid mycobacterial media is acid-fast staining, to confirm that the organisms are indeed mycobacteria. The rate of growth is an important criterion for determining the initial category of the isolate. Rapid growers usually produce colonies within 3 to 4 days after subculture. Mycobacteria can be categorized into three groups based on pigment production. To obtain optimal photochromogenicity, colonies should be young, actively metabolizing, isolated, and well aerated. Although some species turn yellow after a few hours of light exposure, others may take prolonged exposure to light. Most species of mycobacteria produce the intracellular enzyme catalase. Exceptions are certain strains of M. tuberculosis complex and M. gastri.

Illustrated in the upper image is a stained histological section of a bowel showing a heavy inflammatory reaction within which 10 - 12 µm spherical cysts are observed. The lower image is a close up view of one of the trophozoites from a smear preparation of the exudates. There is a distinctive central karyosome, even distribution of the ring of chromatin, and the finely granular cytoplasm. What disease can be presumptively reported? 1) Giardiasis 2) Balantidiasis 3) Amebiasis 4) Cysticercosis

Amebiasis Amebiasis is the correct selection. In the upper image, the size of the spherical shaped parasitic forms can be estimated to be 10 - 12 µm in comparison with the 2 - 3 µm size of the background inflammatory cells. A close view of one of these forms is observed in the lower image. This can be recognized as a trophozoite of Entamoeba histolytica by the distinctive central karyosome, even distribution of the ring of chromatin, and the finely granular cytoplasm providing for a diagnosis of "amebiasis". Giardiasis - the invasive Giardia trophozoites are distinctly oval in shape and have a pair of nuclei on either side of a vertical axostyle, fitted with a central para-basal body. These features resemble a "monkey face". Balantidiasis - the trophozoites of Balantidium coli are kidney shaped and large, approaching 100 µm in diameter, and have a large central dumbbell-shaped nucleus. Cysticercosis - is an infection in which cysts are commonly found in the brain and lungs. Humans become infected after ingesting the ova of Taenia solium contained in contaminated food. Once in the intestine, the ova hatch into larvae that enter the circulation. Although the larvae then permeate fibrous tissue, they do not cause an inflammatory response. Once the larvae lodge in the brain, lungs, or other tissue, they develop into a cyst that transforms into an adult Taenia tapeworm as part of its life cycle.

Which derivative of PCR testing uses short primers not specifically complementary to a particular sequence of a target DNA? 1) Multiplex PCR 2) Nested PCR 3) Arbitrary primed PCR 4) Quantitative PCR

Arbitrary primed PCR testing Arbitrary primed PCR testing uses short (random) primers not specifically complementary to a particular sequence of a target DNA. Multiplex PCR is a method by which more than one primer pair is included in the PCR mixture. Nested PCR involves the sequential use of two primer sets. Quantitative PCR is an approach that combines the power of PCR for the detection and identification of infectious agents with the ability to quantitate the actual number of targets originally in the clinical specimen.

An 18 year old female athlete, who recently competed in a barefoot run for charity in Southern Georgia, presented to her doctor for a routine check-up. Other than complaining of being exhausted and having occasional diarrhea, she was in good health. The doctor ordered a stool for culture and parasite examination. The culture was negative. Numerous suspicious forms as shown in the attached image were seen upon microscopic examination of the stool. The most likely identification is: 1) Hookworm rhabditiform larva 2) Strongyloides rhabditiform larva 3) Artifact 4) Trichinella larva

Artifact Stool samples are known to contain numerous artifacts that may resemble parasites. This image is an artifact. This determination is made by ruling out the other selections as possibilities. Hookworm rhabditiform larva are a typical diagnostic stage along with the egg found in stool. The larva is 250 to 300 µm long, has a long buccal cavity and small inconspicuous genital primordium. None of the diagnostic features are seen in the attached image. Strongyloidesrhabditiform larva are the typical diagnostic stage and found in stool. The larva is 200 to 250 µm long with a short buccal cavity, a large bulb in the esophagus and a prominent genital primordium. None of the diagnostic features are seen in the attached image. Trichinella larva are not typically found in stool. They can be seen on tissue biopsy slides as a coiled encapsulated larva.

This parasitic form, shown in the image to the right, was found in a stool specimen and measured 85 µm in length. Please select the single best answer 1) Schistosoma japonicum egg 2) Paragonimus westermani egg 3) Schistosoma haematobium egg 4) Ascaris lumbricoides egg

Ascaris lumbricoides Ascaris lumbricoides is the correct response. The image shows an unfertilized egg with undifferentiated protoplasm and a distorted outer coat. The unfertilized egg is longer (measuring up to 90 µm in length) as compared to the fertilized egg which measures 60 µm in length and exhibits its typical thick-walled albuminous coat. Schistosoma japonicum eggs measure 55-85 µm by 40-60 µm and are more rounded with a shell and lateral knob with a ciliated miracidium within the egg. Paragonimus westermani eggs have an oval shape with a prominent flattened operculum. The egg has a thickened shell which is opposite the side of the operculum. The egg measures 70 - 110 µm in length. Schistosoma haematobium eggs are elongated and measure from 130-170 µm in length with a ciliated miracidium within the egg.

The photomicrograph shows a close view of erythrocytes three of which are infected. The smear was obtained from a 40-year old forester working in Connecticut who was complaining of malaise and fatigue. Which of the following is the most likely identification? 1) Plasmodium falciparum 2) Plasmodium vivax 3) Babesia microti 4) Borrelia burgdorferi

Babesia microti Babesia microti is the correct selection. The organism can be seen on Wright or Giesma stained thin blood smears. It appears as small, delicate, pleomorphic ring-form trophozoites about 1 to 2 µm in length with a prominent chromatin dot and faintly staining cytoplasm. Mature trophozoites may appear as single, double or the classic tetrad (Maltese cross) formation as is seen here in the right infected cell. Most cases in the US occur in the northeast. Plasmodium falciparum is an incorrect response. P. falciparum might be considered when only the ring forms are observed in any given field of view; however, it does not present in the Maltese cross arrangement characteristic of Babesia. Plasmodium vivax is an incorrect response. P. viviax can be excluded as the infected erythrocytes are typically enlarged, pale staining, and have a cytoplasm with widely dispersed Schuffner's dots. The trophozoite ring forms remain as single ring forms. Borrelia burgdorferi is an incorrect response. B burgdorferi spirochetes are the cause of Lyme Disease that most commonly presents as a skin rash at the site of the tick bite, and later manifests as arthritis or in more severe cases as meningitis or infection of other organs such as the liver. This organism is not observed on blood smears.

All of the following statements are associated with MALDI-TOF mass spectrophotometry used in clinical microbiology, EXCEPT? 1) Bacteria is amplified using a laser. 2) Molecules are measured based on a mass-to-charge ratio. 3) A matrix absorbs energy from the laser and converts it into heat. 4) A spectrum is created.

Bacteria is amplified using a laser. The correct answer is bacteria is amplified using a laser. The bacteria is not amplified. A bacterium is mixed with matrix on a test slide. This is placed on the analyzer which will use a laser to activate the mixture. The matrix is able to absorb the energy from the laser and convert it to heat which will vaporize the surrounding portion of the sample mixture. The vaporized molecules move through a vacuum space at different rates, based on their mass-to-charge ratio, which is determined by their arrival at the detector. These arrival times are then plotted onto a spectrum which is electronically compared with a database to determine the best match and identify the bacterium.

Illustrated in the photomicrograph is a relatively small pear-shaped trematode ovum, averaging 25 - 35 µm, as might be microscopically observed in a stool sample preparation. The shell is smooth and slightly thickened. Distinctive is the shouldered operculum at the cephalic end of the shell and a small knob at the posterior margin (arrows). Developing larvae at varying stages are observed internally. The adult flukes are most commonly found within the liver bile duct system and may cause obstruction and intermittent jaundice. What is the best presumptive identification? 1) Paragonimus westermani 2) Trichuris trichiura 3) Clonorchis sinensis 4) Fasciola hepatica

Clonorchis sinensis Clonorchis sinensis is the correct selection. Key identifying features of the ovum is its relatively small size (average 30 µm), a smooth, slightly thickened shell with a distinct shouldered operculum at the cephalic end. A small knob may be observed at the posterior end. Although the adults occupy the liver bile ducts, the diagnosis of clonorchis infections is most commonly made by observing these distinctive ova in stool specimens. Paragonimus westermani is an incorrect response. Paragonimus ova are large, ranging between 80 -100 µm, and have a smooth, thick shell with a prominent shouldered operculum at the cephalic end, distinct from the ova of Diphylobothrium latum that are of similar size but with a flat, inconspicuous operculum. A posterior knob is not seen on the of P. westermani. Trichuris trichiura is an incorrect response. Trichuris ova average 40 µm, are barrel shaped, have a smooth thick shell. Distinctive is a pair of refractile hilar plugs projecting from each narrow end. Fasciola hepatica is an incorrect response. Fasciola ova are comparatively large, measuring up to 120 µm, and have a thin shell with an indistinct non-shouldered operculum at one of the narrow margins. Internal embryonic cleavage extends to the inner shell.

Although care should be taken when working with all fungus cultures in the laboratory, personnel are particularly prone to develop laboratory acquired infections from the inhalation of airborne species of: 1) Blastomyces dermatitidis 2) Coccidioides immitis 3) Sporothrix schenckii 4) Histoplasma capsulatum

Coccidioides immitis Infection with Coccidioides immitis is acquired through the inhalation of infective arthroconidia. The delicate nature, small size and ease of aerosolization of the arthroconidia of Coccidioides immitis, separated by empty space, makes this species the most likely to result in laboratory acquired infections. Because the arthroconidia are so small and light, they more easily reach the alveolar spaces when inhaled than the conidia of other species. Particularly dangerous are older, mature cultures where spores are highly concentrated and extremely fragile. Although laboratory infections with the other fungal species listed in this exercise are possible, the spores do not become as easily aerosolized and present a less likely event. Of more probability is the direct inoculation of skin through broken glass or penetration of a needle. Blastomyces dermatitidis is believed to begin as a respiratory infection following inhalation of conidia following exposure to contaminated soil or wood. Its spores are not as readily aerosolized. Sporothrix schenckii infection is usually acquired by traumatic contact with thorns or splinters from contaminated vegetation. Histoplasma capsulatum infection occurs after inhalation of conidia from infective structures in the environment. Cases occur in those working in or cleaning out chicken coops, roosting areas of other birds, and barns, as well as in caves due to contact with bat guano.

_______________ is predominantly associated with skin and soft tissue infections (SSTIs), such as abscesses, cellulitis, folliculitis and impetigo. 1) Healthcare-associated MRSA 2) Community-acquired MRSA 3) Escerichia coli 4) Neisseria meningitidis

Community-acquired MRSA Community-acquired MRSA infections are predominantly skin and soft tissue infections (SSTIs), such as abscesses, cellulitis, folliculitis, and impetigo. Outbreaks can occur that are typically seen in communities such as prisons, day care centers, and military recruits. Incidence of infection in pediatric patients is increasing due to these outbreaks. The strain responsible for these infections is called USA300 MRSA and is now being associated with significant bacteremia as well. Healthcare-associated MRSA occur mostly in the elderly population and include infections such as bacteremia and pneumonia. Escherichia coli is able to cause abscesses, cellulitis, folliculitis and impetigo, but it is most commonly associated with urinary tract infections (UTI) Neisseria meningitidis does not typically cause abscesses, cellulitis, folliculitis or impetigo. It is most commonly associated with bacterial meningitis.

The black colonies growing on the surface of this cystine-tellurite agar plate, with dark brown halos surrounding the colonies, is one of the characteristics of: 1) Bordetella pertussis 2) Bordetella parapertussis 3) Legionella pneumophila 4) Corynebacterium diphtheriae

Corynebacterium diphtheriae Cystine-tellurite or Tinsdale agar is used for the isolation of Corynebacterium diphtheriae. Corynebacteria colonies are black due to reduction of tellurite. Further differentiation may be made from observation of brown halos surrounding the black colonies, a result of cystinase activity. In addition to Corynebacterium diphtheriae, cystinase activity is also characteristic of C. ulcerans and C. pseudotuberculosis. Isolation of Bordetella pertussis and Bordetella parapertussis may be achieved with selective enrichment medium such as Bordet-Gengou or Regan-Lowe. Isolation of Legionella pneumophila may be achieved using buffered charcoal yeast extract (BCYE) with L-cysteine.

A 54-year-old Finnish male presented at the local clinic with abdominal pain, weight loss, overall weakness and digestive discomfort. Patient history revealed that the man's diet was rich in raw fish. A complete blood count (CBC) was performed and revealed macrocytic anemia. A stool for parasitic examination was ordered. This suspicious form was seen upon initial screening of the sample. It measures 70 µm by 48 µm. This patient is most likely suffering from an infection with: 1) Clonorchis sinensis 2) Paragonimus westermani 3) Fasciolopsis buski 4) Diphyllobothrium latum

Diphyllobothrium latum Diphyllobothrium latum, the freshwater broad fish tapeworm, is the largest human tapewom. The intermediate hosts of D. latum include crustaceans and freshwater fish. Infection is obtained through the consumption of fresh water fish. The parasite is identified when eggs are visualized in a wet preparation of a fresh stool sample. The eggs are ovoid, operculated (have a lid), with a terminal knob, and often appear yellow-brown. Eggs are approximately 58-75 µm x 40-50 µm in size and pass in abundance in the stool. Eggs are sometimes confused with those of Paragonimus. Clonorchis sinensis, known as the Chinese liver fluke, requires a freshwater snail as an intermediate host. Eggs of C. sinensis are identified in fresh stool using a sedimentation method and a wet mount with or without iodine. Eggs are 28-30 µm x 14-18 µm. They have shouldered opercula with a small knob at the end opposite the operculum and are yellow-brown in color. Paragonimus westermani is the most common and widely distributed lung fluke. Freshwater snails are required as an intermediate host. Eggs are found in stool after adult worms produce eggs in the lungs, which are then expectorated and swallowed. P. westermani eggs measure 80-120 µm x 45-60 µm and are operculated with opercular shoulders, thick shells, and are brownish-yellow in color. They may be misidentified as D. latum, but maybe differentiated by the opercular shoulders and thickened shell and the end opposite the operculum. Fasciolopsis bruski is an intestinal trematode that requires a snail as an intermediate host. Infections are acquired by ingestion of raw or undercooked water plants. The eggs of F. bruski are oval and elongated, transparent, and yellow-brown with an operculum at one end. Eggs are large and measure 130-140 µm x 80-85 µm. Eggs are indistinguishable from those of Fasciola hepatica.

What cestode, pictured here, resides in multiple tissues of the human body? 1) Fasciola hepatica 2) Paragonimus westermani 3) Clonorchis sinensis 4) Echinococcus granulosus

Echinococcus granulosus Echinococcus granulosus is the only cestode that takes up residence in various tissue sites in the body by producing complex miniaturizations of itself. When the eggs are ingested, the oncosphere penetrates the lining of the intestines allowing it to migrate throughout the body. The development of the cyst depends on which organ it attaches to, and symptoms vary according to the organ affected. Fasciola hepatica is not a cestode, but a liver fluke (trematode). The larvae travels through the intestinal wall and peritoneal cavity, and lives in the gallbladder and biliary passages. There are very few symptoms because infections are minimal. Paragonimus westermani is not a cestode either, but a lung fluke. Infection occurs when the cyst travels through the small intestine to the peritoneal cavity. The adults develop in the lung and live in capsules in the bronchioles. Clonorchis sinensis is not a cestode, but also a liver fluke. It is very similar to F. hepatica because causes mild infections and lives in the bile ducts.

A stool was received in the laboratory for parasitic examination on a 49-year-old female who just returned from missionary work in numerous developing countries around the world. The patient had been suffering from mild diarrhea over the previous two weeks. This suspicious form was seen. Notice in the image, five nuclei are visible. Upon focusing between two focal planes, a total of eight nuclei were observed. It measures 25 µm. What is the identification of this form? Image provided courtesy of the CDC 1) Entamoeba hartmanni 2) Entamoeba coli 3) Entamoeba histolytica 4) Artifact

Entamoeba coli This case is an example of a patient infected with an intestinal ameba, Entamoeba coli. Although the pathogenicity of Entamoeba coli has been questioned, to date it is generally considered a commensal in the intestinal tract transmitted by ingesting cysts in food or water contaminated with feces. Examination of forms seen in specimens, specifically the nuclear characteristics and appearance of the cytoplasm, including inclusions when appropriate, are important in order to successfully identify what is being seen. E. coli is the only species of Entamoeba found in humans that has a cyst stage containing more than four nuclei. Another characteristic that aids in identification is the size of the cyst. E. coli cysts are typically larger than cysts of the other intestinal amoebae, ranging from 10 to 35 µm. Entamoeba hartmanni is considered a nonpathogen. The cysts are small, measuring 5 to 10 µm and contain four nuclei. Entamoeba histolytica is a major pathogen. It can cause colitis and hepatic abscesses due to its ability to erode the intestinal wall and travel to the liver through the circulatory system. The E. histolytica cyst measures 10 to 20 µm and contains four nuclei.

Which organism will cause amebiasis and is considered pathogenic? 1) Entamoeba coli 2) Entamoeba dispar 3) Entamoeba histolytica 4) Entamoeba hartmanni

Entamoeba histolytica Invasive intestinal amebiasis has various clinical forms, all of which are generally acute: amoebic diarrhea without dysentery, dysentery (bloody diarrhea) or colitis, ameboma, and amoebic liver abscess. Amebiasis is caused by infection with the true pathogen, Entamoeba histolytica. Entamoeba coli typically is considered nonpathogenic and will not cause disease. Entamoeba dispar produces no intestinal symptoms and in not invasive in humans (nonpathogenic). Entamoeba hartmanni is considered nonpathogenic and does not cause disease.

Which one of the following organisms is typically urease negative: 1) Klebsiella 2) Escherichia 3) Proteus 4) Haemophilus

Escherichia The urease test determines whether an organism can hydrolyze urea, which releases ammonia. Urease testing may be used to differentiate between members of Enterobacteriaceae. E. coli does not produce urease. Clinically significant species of Klebsiella, such as K. pneumoniae and K. oxytoca may produce urease, along with species of Proteus. Urease, along with indole and ornithine decarboxylase, may be used to biotype Haemophilus species.

Illustrated in the top photograph is a circumscribed colony after 3 days of incubation at 30o C. The consistency is cottony to wooly and pigment ranges from gray to deep brown. A light pigmented outer apron is seen at the margins of growth. The reverse is dark brown to black, resulting from a darkly pigmented mycelium. As the colony morphology is not species specific, the identification can be made by the appearance of the macroconidia observed in a stained tease mount made from the surface of the colony as observed in the bottom photomicrograph. From these observations, select from the multiple choices the name of this fungus. 1) Curvularia species 2) Exserohilum species 3) Nigrospora species 4) Bipolaris species

Exserohilum species Exserohilum species is the correct response. The conidia are distinctive in being long and pencil-shaped, with a large number of cells (8 - 10) separated by transverse septa. Of particular note is the extended, prominent protruding extension from the hilar cells, from which the genus name "Exserohilum" is derived. Curvularia species is distinctive for the twisted geniculate conidiophores from which are produced large, brown-stained macroconidia divided by transverse septa into only 4 to 5 cells. Also distinctive is the overgrown cell within each of the conidia giving a curved shape. Nigrospora species conidia are single cells devoid of septations. They are relatively large, subglobose in shape, smooth and jet black, being borne atop the tips of inflated, urn-shaped conidiophores. Bipolaris species macroconidia are long and elliptical to oval, each with a thick, smooth wall, separated into multiple equal sized cells less in number than those of Exserohilum species. The macroconidia are produced in loose sympodial clusters at the tips of long, twisted "knee bend", septate conidiophores. A protruding extension from the hilar cell is not produced.

This parasite is found worldwide, particularly in areas in which sheep and cattle are raised. The natural host for the completion of this parasite's life cycle is the sheep. Humans often serve as accidental hosts. Which of the following conditions may be associated with the presence of this stool parasite from a human measuring 130-150 µm long by 60-90 µm wide? 1) Paragonimiasis 2) Fasciolopsiasis 3) Fascioliasis 4) Ascariasis

Fascioliasis The disease caused by Fasciola hepatica is called Fascioliasis. The egg shown here resembles that of either Fasciola hepatica or Fasciolopsis buski. The eggs of both flukes are identical and are considered indistinguishable. They both consist of an oblong undeveloped miracidium equipped with a distinct operculum. However, the upper limit of size is considered Fasciola hepatica. Both of these parasites are transmitted through ingestion of raw infected water plants. Additionally, the geographic distribution of Fasciola hepatica is worldwide and the natural host for completion of the life cycle is the sheep. Recovery of the adult flukes is necessary to confirm the identification between the two organisms. The parasitic organism Paragonmus westermani causes the disease paragonimiasis. It is transmitted through the ingestion of undercooked crayfish or crabs and occurs in several areas of the world: Asia, Africa, India, and South America. Once ingested this fluke migrates through the intestinal wall eventually going into lung tissue. The eggs/ova of Fasciolopsis buski resemble Fasciola hepatica but are slightly smaller in size, and the location of where the adult worms take up residence in the human host differs: F. buski in the small intestine and F. hepatica in the bile ducts. The parasite Fasciolopsis buski causes the disease fasciolopsiasis. Additionally the geographic distribution of F. buski is limited to areas of the Far East and it has animals such as rabbits, pigs, and dogs as the reservoir hosts. All of these diseases are caused by trematodes except for ascariasis which is a worldwide infection caused by the large intestinal roundworm/Nematode of Ascaris lumbricoides. This parasite's egg is smaller in size than the other answer choices presented. Infection occurs through direct ingestion of the ova usually as a result of using human feces as a fertilizer.

Illustrated in this photograph is a blood agar plate inoculated with a beta hemolytic streptococcus obtained from a throat culture of a patient with acute pharyngitis. A 0.04 µg bacitracin (left) disk and a SXT disk (right) had been placed in the areas of streaking. The reactions observed allow the most likely presumptive identification of a streptococcus belonging to Lancefield group: 1) Group A 2) Group B 3) Group C 4) Group G

Group A The correct answer is group A. The zone of growth inhibition around the bacitracin (left) disk is presumptive evidence for a beta hemolytic streptococcus, group A. Further confirmation is the resistance to SXT (right). Group A streptococci are often recovered from throat infections; therefore, he reactions shown in this photograph along with the site of recovery allows the most likely report of, "beta-hemolytic streptococcus, presumptive group A." Group B streptococci demonstrate resistance to bacitracin. While there are occasional strains that may show sensitivity to bacitracin, group B streptococci would be an extremely rare isolate from throat cultures. It should be remembered that approximately 10% of group C streptococci may produce narrow zones of hemolysis around the bacitracin (left) disk, is also susceptible to SXT (right). Group G streptococci may produce a narrow zone of hemolysis around the bacitracin, but are susceptible to SXT, differentiating them from group A streptococci.

A positive CAMP reaction and hippurate hydrolysis are key in identifying which beta-hemolytic streptococci? 1) Group B 2) Group A 3) Group F 4) Viridans

Group B Of the beta-hemolytic streptococci, only S. agalactiae (Group B) is positive for CAMP and hippurate hydrolysis. The other beta-hemolytic streptococci are negative for both tests which includes S. pyogenes (Group A) and S. aginosus group (Group F). Viridans streptococci are either alpha or gamma-hemolytic and are usually negative for both tests.

From the multiple choices below, select the most appropriate culture medium for optimum recovery and differentiation of Salmonella and/or Shigella. 1) Columbia colistin-nalidixic acid (CNA) agar 2) Thiosulfate citrate bile salts sucrose (TCBS) agar 3) Hektoen enteric (HE) agar 4) Sabouraud dextrose agar

Hektoen enteric (HE) agar Hektoen enteric agar is used for recovery of Salmonella and Shigella. It contains bile salts that inhibit Gram positive organisms and most Gram negative enteric flora. Fermentation of lactose, sucrose, or salicin results in an acidic pH, and color change to orange or pink. Salmonella and Shigella appear blue to blue-green from lack of fermentation. Colonies of Salmonella species that produce hydrogen sulfide will contain black centers from sodium thiosulfate and ferric ammonium citrate present in the medium. Columbia CNA agar is a blood agar base containing colistin and nalidixic acid, allowing for selectivity of Gram positive organisms in mixed specimens. Thiosulfate citrate bile salts sucrose agar selects for Vibrio species in mixed stool specimens. TCBS contains sucrose and a blue indicator for detection and differentiation of fermentation. TCBS also has a high pH for suppression of other intestinal bacteria, bile salts to inhibit Gram positive bacteria, sodium citrate and sodium thiosulfate to support halophilic Vibrio species and to detect hydrogen sulfide. Sabouraud dextrose agar is used for the cultivation of fungi. Selectivity is based on the antimicrobial content (chloramphenicol, gentamicin, and tetracycline).

A track star was in the locker room after a competition when one of his teammates noticed light brown circular lesions on his upper back. The agent MOST likely responsible for this condition is: 1) Candida albicans 2) Tinea versicolor 3) Sporobolomyces spp. 4) Malassezia furfur

Malassezia furfur. M. furfur is most likely as it causes tinea versicolor (brown lesions on back). Malassezia furfur can be found as normal flora on healthy skin of many individuals and in patients with clinical disease. The organism is found in both the yeast stage and the filamentous form. Factors that may lead to the infection with Malassezia furfur as opposed to normal flora include: warm, humid environments, immunosupression, Cushings disease, etc. Candida albicans and Sporobolomyces spp. are not known to cause tinea versicolor. Tinea versicolor is a condition, not an organism.

This suspicious form, found in stool, measures 54 µm by 32 µm. What is the identification? 1) Taenia egg 2) Echinococcus egg 3) Hymenolepis egg 4) Pseudoparasite

Hymenolepis egg The image shows an egg of Hymenolepis. Hymenolepis eggs have a characteristic embryo in the center with three pairs of of hooks. Surrounding the embryo is an embryophore and a capsule. H. diminuta averages 55 by 85 µm and H. nana typically measures 45 by 38 µm in size. Taenia eggs are similar to Hymenolepis as they also have a central embryo with three pairs of hooklets. However, the embryophore of Taenia has radial striations and is typically a yellow/brown color. Additionally, Taenia eggs are smaller, averaging 33 by 23 µm in size. The diagnostic stage of Echinococcus infections is not the egg form, but rather the larval stage. The larval stage is characterized by a hydatid cyst, which is found in infected tissue. Pseudoparasite means a false or fake parasite, such as matter in the sample that resembles a parasite, but is not one. Based on the very round edges and visible internal structures, this is an egg, specifically a Hymenolepis egg.

Beyond direct detection, what molecular method listed below is being widely used in clinical laboratories across the United States and allows for rapid detection of a bacteria from an isolated colony? 1) MALDI-TOF 2) Nanotechnology 3) Next generation sequencing 4) Pyrosequencing

MALDI-TOF Matrix assisted laser desorption time of flight (MALDI-TOF) can be used once a colony is isolated on solid media. A portion of the colony is placed on a metal target and a matrix is applied to allow the colony proteins to be cocrystallized. This target is then loaded into the instrument where a laser is used to transfer protons to the colony/matrix mixture which vaporizes the proteins. The proteins are then separated by the mass analyzer based on their mass-to-charge ratio creating a unique set of signals (mass spectrum). Once the target is entered into the instrument, it will take between 30 to 60 seconds to generate the mass spectrum. Nanotechnology is creating materials that can be used in many arenas, including medicine. This technology is gaining popularity, but due to its specificity and cost, it is not being widely used. Next generation sequencing is combining high throughput and accuracy for detecting microorganisms. Unfortunately, this technology is both expensive and time consuming, but is being used more often in the health department setting. Pyrosequencing is a newer sequencing technology that is being used to identify microorganisms and antimicrobial resistance genes. This technology is also expensive and time consuming, thus it is not seen often in clinical laboratories.

Tiny translucent gray colonies were recovered on blood agar from a patient with purulent conjunctivitis. Gram negative diplococci were seen on Gram stain. The preparation illustrated in this photomicrograph was obtained from colonies growing at the margin of inhibition of a 10 µg/mL penicillin disk. The most likely identification is: 1) Neisseria gonorrhoeae 2) Moraxella lacunata 3) Neisseria sicca 4) Acinetobacter baumannii

Moraxella lacunata Moraxella lacunata is the correct answer because performing Gram stains on a smear prepared from the margin of growth inhibition by a 10 µg/ml penicillin disk in a disk diffusion test is a valuable method for differentiating Moraxella species from Neisseria species. This will help determine if the organism in question is a true cocci or has elongated. The bacterial cells of Moraxella species, under the influence of the sub-optimal concentrations of penicillin at the outer margin of growth inhibition, form the elongated, pleomorphic filaments seen here. The bacterial cells of Neisseria species, under these same conditions, remain spherical. Neisseria gonorrhoeae is incorrect because the Gram stain demonstrates elongated, pleomorphic filaments rather than true cocci. Neisseria sicca is incorrect because the Gram stain demonstrates elongated, pleomorphic filaments rather than true cocci. Acinetobacter baumannii is a true Gram negative rod that can demonstrate a Gram negative coccobacilli morphology. Acinetobacter species can resist decolorization causing them to appear as gram positive instead of gram negative. The organism can also be mistaken for a Neisseria species due to the coccobacilli morphology when staining gram negative. Acinetobacter species are oxidase negative while Neisseria species are oxidase positive.Moraxella lacunata

The nitrate reduction test is used to determine the ability of an organism to reduce nitrate. The Enterobacteriaceae group can reduce nitrate. Based on this test, what is the end product for which nitrate is reduced? 1) CO2 2) NH3 3) O2 4) N2

N2 N2 is the correct answer because nitrate can be reduced to nitrite or nitrogen gas. Denitrification occurs by which inorganic nitrate (or nitrite) is reduced all the way to nitrogen gas. This test can be read for the presence or absence of three metabolic products: nitrogen gas, nitrate (NO3), and nitrite (NO2). CO2 is incorrect because this results from the decarboxylation of amino acids resulting in the production of carbon dioxide. NH3 is incorrect because this byproduct results from the action of urease on urea, which liberates NH3. O2 is incorrect because the release of O2 occurs with the catalase test. Hydrogen peroxide is reduced to oxygen and water.

The human infections related to the 60 X 40 µm ovum, as illustrated in the image, are reported by the World Health Organization to involve several million people worldwide. Symptoms vary from nausea and diarrhea in patients with light infection, to peripheral edema, anemia, dehydration, and congestive heart failure in those with heavy infections. Select the most likely genus/species identification. 1) Enterobius vermicularis 2) Necator americanus 3) Trichuris trichiura 4Ascaris lumbricoides

Necator americanus Necator americanus is the correct response. Characteristic of the genus is the thin, smooth, transparent shell with the developing internal yolk sack retracting from the shell leaving an open clear space. Enterobius vermicularis is an incorrect response. E. vermicularis ova are oval in outline and asymmetrical, with one side flattened and simulating a deflated football. The shell is slightly thickened, smooth, and transparent. A well-developed larva is often observed within the ovum when mature. Trichuris trichiura ova are among those most easy to recognize. They are distinctly barrel-shaped, with characteristic retractile, protruding convex, hyaline polar plugs at either end. The shell is smooth, but relatively thick with an internal developing embryo reaching the inner surface without leaving a space. Ascaris lumbricoides is an incorrect response. Ascaris ova are characteristically yellow-brown (bile-stained), oval or spherical, and have a thick, transparent, hyaline shell, covered by an albuminous coat. Fertilized eggs can be recognized by the cleavage of the internal yolk. A distinct larva may be seen in later stages of development.

The bacterial species shown in this composite photograph of a chocolate agar plate and accompanying gram stain, recovered from an autopsy blood culture of a patient dying with Waterhouse Friderichsen syndrome is most likely: 1) Moraxella catarrhalis 2) Neisseria gonorrhoeae 3) Neisseria meningitidis 4) Acinetobacter baumannii

Neisseria meningitidis Of the bacterial species listed in this exercise, Neisseria meningitidis is the primary cause of Waterhouse Friderichsen syndrome. The small, smooth, gray to white, opaque colonies (illustrated on the chocolate agar plate in the upper frame) are consistent with N. meningitidis and the gram stain (lower frame) shows gram-negative diplococci. Waterhouse Friderichsen syndrome is a fatal, disseminated form of meningococcemia in which microthrombi from throughout the vasculature secondary to disseminated intravascular coagulation (DIC), with the terminal event being acute bilateral hemorrhage into the adrenal glands. Other bacterial species, notably certain strains of Staphylococcus aureus, Pneumococcus pneumoniae and Haemophilus influenzae, on occasion can cause Waterhouse Friderichsen syndrome. However, N. meningitidis is by far and away the leading cause. Moraxella catarrhalis is an opportunistic pathogen that normally inhabits the upper respiratory tract. It may cause pneumonia, otitis media, and sinusitis. It rarely disseminates to cause bacteremia and meningitis. The organism is similar to Neisseria meningitidis as it is a Gram-negative diplococcus that grows as a small, brown, opaque colony on chocolate agar. It is not a cause of Waterhouse Friderichsen syndrome. Neisseria gonorrhoeae is never normal flora. It is a sexually transmitted disease that infects the mucous membranes of the genital tract, oropharynx, anorectal area, and conjunctiva. It may become a disseminated infection resulting in bacteremia, arthritis, and pelvic inflammatory disease (PID) but it is not a known cause of Waterhouse Friderichsen syndrome. It resembles Neisseria meningitidis as it is a gram-negative diplococcus and it appears as small, smooth, gray to white, translucent colonies on chocolate agar. Neisseria sicca is normal flora of the upper respiratory tract. It typically does not cause infections but may cause opportunistic bacteremia, endocarditis, and meningitis. It is not a known cause of Waterhouse Friderichsen syndrome. It is a gram-negative diplococcus that grows well on chocolate agar as a large, non-pigmented, wrinkled, dry colony.

A 17-year-old female went to her doctor complaining of diarrhea. A stool specimen was submitted for cul ture and parasite examination. The culture was reported out as "no enteric pathogens isolated." These suspicious forms were seen on wet preparation and permanent stain. The form shown in the upper left measured approximately 32 µm, and appeared to have a slow, non-directional movement. The other forms measured approximately 20-22 µm. The patient most likely has which of the following conditions? 1) Amebiasis 2) None; considered non-pathogenic 3) Traveler's diarrhea 4) Giardiasis

None; considered non-pathogenic Depicted in the upper left is a trophozoite form, along with three cyst forms of Entamoeba coli. The mature cyst of Entamoeba coli has eight nuclei. Although the intestinal amoeba is considered a non-pathogen, it is important to identify as it indicates contaminated food or drink has been consumed. Practitioners should be flagged when such parasites are seen that it is possible that pathogenic ones, such as Entamoeba histolytica, may also be present. Thorough examination of the specimen is recommended prior to turning out the final results. Amebiasis is caused by Entamoeba histolytica. Traveler's diarrhea is commonly caused by strains of Escherichia coli. Giardiasis, or beaver fever, is caused by Giardia duodenalis.

Identify the species of dimorphic fungi in its yeast form as observed under the microscope. 1) Blastomyces dermatitidis 2) Paracoccidioides brasiliensis 3) Sporothrix schenckii 4) Coccidioides immitis

Paracoccidioides brasiliensis Paracoccidioides brasiliensis produces large yeast cells, approximately 15-30 µm in diameter, with multiple buds attached by narrow necks, giving the appearance of a "mariner's wheel." The yeast form of Blastomyces dermatitidis is a thick-walled yeast cell, measuring 8-15 µm in diameter, that characteristically produces a single bud attached by a broad base. The yeast forms of Sporothrix schenckii are elongated cells that have been called "cigar bodies." Coccidioides immitis does not produce a yeast form in laboratory culture; rather, is identified in stained tissue sections by the production of varying sized spherules, ranging from 30-60 µm in diameter at maturity. The larger, more mature spherules may contain spherical endospores, making identification from direct examination difficult, as Coccidioides species may resemble other dimorphic fungi.

All of the organisms are listed with a common site of recovery for that parasite, EXCEPT: 1) Giardia lamblia - duodenal contents 2) Trichuris trichuria - stool 3) Plasmodium falciperum - ocular secretions 4) Paragonimus westermanni - sputum

Plasmodium falciperum - ocular secretions. Plasmodium falciperum is commonly found in the peripheral blood and can be recovered from liver biopsies as well. It is part of the blood and tissue flagellates group. It is known to cause severe CNS complications where a patient will complain of severe headaches, will be confused and will eventually lapse into a coma. Giardia lamblia is found in both stool and duodenal contents. Trichuris trichuria is found in stool samples, as the eggs are shed in the feces. Paragonimus westermanni is found in sputum, as it is known to inhabit the lungs.

The following is true of the disk diffusion method of measuring antimicrobial sensitivity: 1) Provides a semi-quantitative MIC value 2) Provides only an interpretation value of sensitive, intermediate, or resistant correlated to MIC values 3) Requires minimal standardization 4) Commonly automated method of antimicrobial susceptibility testing

Provides only an interpretation value of sensitive, intermediate, or resistant correlated to MIC values The disk diffusion method of antimicrobial susceptibility testing provides an interpretation value of sensitive, intermediate, or resistant. The zone of inhibition is correlated to the broth or agar dilution MIC values. The disk diffusion method of antimicrobial susceptibility testing does not provide a semi-quantitative MIC value. The zone of inhibition surrounding the disk is measured in millimeters and correlated to the broth or agar dilution MIC value giving interpretations of sensitive, intermediate or resistant. The disk diffusion method requires standardization of pH, agar depth, inoculum concentration, disk concentration, cation concentration, and more. The disk diffusion method of antimicrobial testing is most frequently manually interpreted. Automation has been developed but it is not as commonly used as the automated MIC methods.The following is true of the disk diffusion method of measuring antimicrobial sensitivity: The correct answer is highlighted below Provides a semi-quantitative MIC valueProvides only an interpretation value of sensitive, intermediate, or resistant correlated to MIC valuesRequires minimal standardizationCommonly automated method of antimicrobial susceptibility testing

A suspected nocardioform bacterial species was recovered from sputum. The isolate produced mucoid colonies with pink pigment after 4 days on SBA, as shown in the associated image. Staining demonstrated diphtheroid, Gram positive rods with a few branching filaments. It also stained partially acid fast. The most likely identification is: Image courtesy of CDC: https://phil.cdc.gov/Details.aspx?pid=1553 1) Rhodococcus equi 2) Corynebacterium pseudotuberculosis 3) Klebsiella pneumoniae 4) Cutibacterium (Propionibacterium) acnes

Rhodococcus equi Rhodococcus equi is classified as an aerobic actinomycete and nocardioform bacteria. Colonies of R. equi produce salmon-pink pigment, especially with extended incubation. Growth on SBA may resemble lactose-fermenting Klebsiella on MAC. Corynebacterium pseudotuberculosis is a Gram positive rod, diphtheroidal in arrangement, and does not form filaments. It produces small, yellowish-white colonies on SBA. Klebsiella pneumoniae is Gram negative rod, and does not produce filaments or stain acid fast. It grows as mucoid, gray-white colonies on SBA. Cutibacterium (Propionibacterium) acnes is a Gram positive, anaerobic rod with diphtheroid-like arrangement that does not produce branching filaments like Cutibacterium (Propionibacterium) propionicus. On anaerobe blood agar, colonies are circular, entire, convex, and glistening. It is often a contaminant from skin, but may cause more serious infection.

This suspicious form, found in urine, measures 120 µm by 50 µm. What is the appropriate identification? Please select the single best answer 1) Schistosoma japonicum 2) Pseudoparasite 3) Trichomonas vaginalis 4) Schistosoma haematobium

Schistosoma haematobium The only egg, as seen in the photograph, typically recovered in urine samples is that of Schistosoma haematobium. This organism is characterized by its distinct shape, particularly the presence of a large terminal spine opposite a rounded end. The standard method of diagnosis is by the detection of characteristic eggs in feces for Schistosoma japonicum which has a small lateral spine. Since this is showing an actual egg form of a parasite, it would not be considered a pseudoparasite. Although Trichomonas vaginalis may be recovered from a urine sample, it is relatively small in size (average length = 13 µm) and assumes the characteristic flagellate trophozoite appearance.

Which of the following organisms is considered a Helminth? 1) Trypanosoma brucei rhodesiense 2) Leishmania braziliensis 3) Entamoeba histolytica 4) Schistosoma mansoni

Schistosoma mansoni Schistosoma mansoni is a blood fluke, or Helminth, that can cause local infections of dermatitis that lasts for 3 days. However, if the larvae travel through the body it can cause fever, malaise and urticaria that can last up to 4 weeks. Trypanosomes brucei rhodesiense is a Protozoa that causes East African sleeping sickness. The infection starts as a tender, red inflammation at bite that progresses to a hemolymphatic disease in about 3 weeks. The infection can then progress to the CNS where it can cause paralysis and eventually death. Leishmania braziliensis is a Protozoa that causes mucocutaneous leshmaniasis that destroys the mucosal lining in the face. This can leave hosts significant damage. Entamoeba histolytica is an intestinal amebae that is usually asymptomatic, but can cause abdominal infections. Asymptomatic patients pose a threat because they are contagious and can pass infection to others.

The Entamoeba histolytica trophozoite is usually characterized by the presence of the following: 1) Small central karyosome in the nucleus, ingested RBCs 2) Ingested RBC's, coarse, uneven chromatin on the nuclear membrane 3) Ingested RBCs, large glycogen vacuoles in cytoplasm 4) Large, blot-like karyosome, ingested WBCs

Small central karyosome in the nucleus, ingested RBCs The Entamoeba histolytica trophozoite is usually characterized by the presence of a small central karyosome and evenly distributed peripheral chromatin. Ingested RBCs can be seen within the cytoplasm. Ingested RBC's, coarse, uneven chromatin is incorrect due to E. histolytica having evenly distributed chromatin. Ingested RBC's, large glycogen vacuole in cytoplasm is incorrect as the glycogen vacuole is typically seen in immature cysts not trophozoites. The question is asking about E. histolytica trophozoites not cysts. Large, blot-like karyosome, ingested WBC's is incorrect as Entamoeba histolytica does not have a blot-like karyosome. Acanthamoeba species will have a blot-like karyosome.

The class of protozoa with no apparent organelles for locomotion is known as: 1) Mastigophora 2) Sporozoa 3) Ciliata 4) Sarcodina

Sporozoa The Sporozoa is a class of parasites that are non-motile. The phylum of non-motile parasites is Apicomplexa. Some of the organisms in this class include the Plasmodium species, Babesia, Cryptosporidium, and Toxoplasma. Mastigophora is a subphylum of organism that are all motile via flagella. Some organisms in this subphylum include Giardia intestinalis and Trichomonas vaginalis. Ciliata is a group of parasites that are motile via cilia. There is only one human parasite in this group, Balantidium coli. Sarcodina is a subphylum of parasites that are motile via pseudopodia, such as the amoebas. Some organisms in this subphylum include Entamoeba species.

Observed on the surface of blood agar are smooth, white, 48-hour-old colonies incubated at 37oC, demonstrating a positive catalase reaction and a negative coagulase reaction. The sample was taken from an intravenous catheter. In the photomicrograph are gram positive cocci distinctly arranged in loose clusters and in tetrad formation. From these observations, select the presumptive identification of this isolate. 1) Staphylococcus aureus 2) Staphylococcus epidermidis 3) Micrococcus luteus 4) Rothia mucilaginosa

Staphylococcus epidermidis Staphylococcus epidermidis is the correct response. Colonies are smooth with a white or gray-white color. Microscopic observation of gram stains reveals gram-positive cocci in small loose clusters, often in tetrad formation. The catalase reaction is positive, and the coagulation reaction is negative. A definitive identification requires further testing. Staphylococcus aureus colonies are smooth and may be cream-colored or yellow, but can be microscopically differentiated from both S. epidermidis and Micrococcus sp. by observing gram-positive cocci that form tight grape-like clusters rather than arranging in loose clusters or in tetrads. Confirmation can be made by demonstrating a positive coagulase reaction. Micrococcus luteus colonies are smooth, but often produce distinctly yellow pigment. Tetrad formation of the gram-positive cocci may be observed microscopically in gram stains. The catalase reaction is positive, and the coagulation reaction is negative. Confirmation of M. luteus can be made with further testing, such as positive modified oxidase, and susceptibility to bacitracin ("A disc"). Rothia mucilaginosa colonies are clear to light pink-gray and are distinctly mucoid and adherent to the agar surface. Rothia spp. may resemble gram positive coryneform bacilli, in pairs or short, filamentous chains, but gram positive cocci are observed when grown in broth. A definitive identification requires further testing. Rothia is also indigenous to the oral cavity and may be recovered in cases of dental plaque infections.

A patient presents with cellulitis and culture growth small beta hemolytic colonies on 5% Sheep blood agar. The gram stain of the organism is included. What is the most likely cause of the infection. 1) Streptococcus pyogenes 2) Staphylococcus aureus 3) Pseudomonas aeruginosa 4) Enterococcus faecalis

Streptococcus pyogenes Streptococcus pyogenes is the causative agent of cellulitis in this patient. S. pyogenes grows as small catalase negative colonies with a small zone of beta hemolysis on 5% sheep blood agar. The Gram stain shows gram positive cocci in chains. If left untreated the patient may progress into streptococcal toxic shock syndrome. Staphylococcus aureus infections may also produce cellulitis in people that can be difficult to treat and sometimes lead to a toxic shock syndrome. However, in the case of this patient, S. aureus is not the cause of infection. S. aureus grow as larger catalase positive cream colored colonies on 5% sheep blood agar with a wide zone of beta hemolysis, sometimes seen as a double zone. The Gram stain for S. aureus show gram positive cocci in clusters. Although Pseudomonas aeruginosa may be seen in soft tissue infections, it is most often associated with cystic fibrosis patients and situations where biofilms are generated. P. aruginosa grows as large oxidase positive beta hemolytic colonies with a green to silver sheen on 5% sheep blood agar. They also grow as non-lactose fermenting colonies on MacConkey agar. The Gram stain for P. aruginosa shows gram negative bacilli. Enterococcus faecalis appear morphologically similar to S. pyogenes by Gram stain; however, E. faecalis grow as small catalase negative gray colonies on 5% sheep blood agar and do not show any hemolysis. E. faecalis may be seen in some soft tissue infections, but does not typically cause cellulitis.

This suspicious form was recovered in stool. What is the identification? 1) Brugia microfilaria 2) Pseudoparasite 3) Taenia solium 4) Strongyloides rhabditiform larva

Strongyloides rhabditiform larva Strongyloides stercoralis rhabditiform larvae are the only such larvae that are routinely found in stool samples. The rhabditiform larvae are the actively feeding stage. Not as noticeable in the image is the short buccal cavity and prominent genital primordium that is characteristic of Strongyloides. These larvae differ from hookworm larvae because hookworm larvae will have a long buccal cavity and small genital primordium. Hookworm larvae are also rarely found in the stool, and instead, the eggs are more commonly seen. Brugia microfilariae are recovered in blood and are not found in stool samples. Brugia is transmitted by different species of mosquitos, depending on the geographical area. The worm lives in the blood and lymphatic system. Psuedoparasite means false or fake parasite, which would indicate this was some type of fecal matter that resembled a parasite but was not one. Based on the defined shape and organism, it is suggestive of a true parasite. Other clues indicate the presence of internal structures. Taenia solium is the pork tapeworm. Typically eggs are seen in the stool of infected patients. However, worms can be seen, but the tapeworms have a different body structure made up of proglottids. The proglottids can be seen in the stool and contain lateral uterine branches that can differentiate the Taenia species.

The rhabditiform larva shown in this image was observed in a mount prepared from a diarrheal stool specimen. This patient also complained of persistent cough and increased shortness of breath. From the list of answer choices below, what presumptive identification can be made? 1) Strongyloides stercoralis 2) Trichuris trichiura 3) Ancylostoma duodenale 4) Necator americanus

Strongyloides stercoralis Strongyloides stercoralis is the correct response. Diagnostic is the short buccal cavity (arrow) as observed in the anterior section of the rhabditiform larva. Not observed in this photograph is the genital primordium that commonly is seen 1/3 the distance from the tail. Trichuris trichiura is an incorrect response. Trichuris larvae hatch soon after ingestion of an infective ovum and develop into adult worms in the small intestine producing eggs that are passed in stool. Ancylostoma duodenale is an incorrect response. The buccal cavities of the rhabditiform larvae of this hookworm species are long differentiating it from the short buccal cavity of Strongyloides. Necator americanus is an incorrect response. The buccal cavities of the rhabditiform larvae of this hookworm species are long differentiating it from the short buccal cavity of Strongyloides.

A patient came into the emergency department with a swollen knee and the physician drained 20 mL of fluid from the patient's knee. What is the name of the fluid collected? 1) Pleural fluid 2) Ascitic fluid 3) Synovial fluid 4) Pericardial fluid

Synovial fluid Synovial fluid is correct as fluid collected from joints is referred to as synovial fluid. Fluid can accumulate in the joints due to infections or inflammation, such as arthritis. Pleural fluid is incorrect because pleural fluid is found in the pleural space (lining between the chest wall and lungs). The fluid collected will be sent to the laboratory for testing, such as cell count, total protein, glucose, and culture. Fluid collected from the thorax can also be called thoracentesis, pleural, or empyema fluid. . Ascitic fluid is incorrect because this fluid is collected from the abdominal cavity and is also referred to as paracentesis or peritoneal fluid. During an inflammatory or infectious process, such as liver disease or trauma, fluid can accumulate within the peritoneal cavity. Once collected the fluid can be sent to the laboratory for testing. Such tests are amylase, protein, albumin, cell count, culture, and cytology. Pericardial fluid is incorrect because this fluid surrounds the heart. If fluid accumulates around the heart in the pericardial space, the pericardial fluid will be drained and sent to the laboratory for testing. Causes for increased pericardial fluid is pericarditis or myocarditis due to viruses, bacteria, fungi, or parasites.

What is this parasite that resides in the human intestine? 1) Taenia solium scolex 2) Hymenolepis nana scolex 3) Dipylidium caninum scolex 4) Diphyllobothrium latum scolex

Taenia solium scolex Taenia solium is the pork tapeworm, and when humans ingest raw or undercooked meat the scolex is passed into the small intestines where it attaches and matures to an adult worm. The Taenia soliumscolex is identified by the presence of four suction-cup like suckers and the presence of a fleshy rostellum with hooklets. Hymenolepis nana commonly infects children, because an intermediate host is not required. The scolex in the larval stage is found in the tissues of the intestines, and is characterized by four small suckers and a rostellum with spines. Dipylidium caninum is the dog tapeworm, and humans are accidental hosts usually by ingesting the fleas containing the larval stage. Infections are generally asymptomatic. Diphyllobothrium latum is the fish tapeworm. The scolex resides in the flesh of the fish, and is released in the intestines where it develops into an adult tapeworm. This scolex is 2-3 mm long, elongated with two sucking grooves.

This photograph is of an isolate from a stool culture on a 4-year-old child with gastroenteritis. The medium contains cefsulodin, irgasan, novobiocin, bile salts, mannitol, neutral red, and crystal violet. Further identification tests will most likely identify this isolate as: 1) Salmonella spp. 2) Shigella spp. 3) Campylobacter jejuni 4) Yersinia enterocolytica

Yersinia enterocolytica Yersinia enterocolytica is the correct answer. Cefsulodin-irgasan-novobiocin (CIN) agar is selective for Y. enterocolytica. It forms red centered colonies because the mannitol is fermented causing a change in pH of the neutral red indicator. Salmonella, Shigella, and Campylobacter will not grow on CIN medium. Salmonella and Shigella will appear as non-lactose fermenting colonies on MacConkey agar and highly selective media, such as Hektoen enteric (HE). HE medium also includes ferric salts which will show black centers in the colonies if hydrogen sulfide gas is produced. Salmonella will grow as blue-green colonies (a non-lactose fermenter) with black centers on HE and Shigella will produce blue-green colonies without black centers on HE. Stool specimens should also routinely include screening for Campylobacter spp. In order to screen for Campylobacter, Campy blood agar - a selective, enriched agar - is inoculated and incubated at 42 degrees Celsius in a microaerophilic atmosphere.

The 40 µm in diameter ovum illustrated in the photograph may be observed in the stool specimens of individuals presenting with minimal abdominal discomfort and low grade diarrhea. Pairs of hooklets are observed internally in the absence of an internal membrane. Select from the multiple choice answers the presumptive identification of this cestode ovum. 1) Hymenolepis diminuta 2) Diphyllobothrium latum 3) Taenia spp. 4) Dipylidium caninum

Taenia spp. Taenia species is the correct response. Characteristics of Taenia ova include the spherical outline and smooth, thick shell with distinctive radial striations. Although the pairs of hooklets observed internally might suggest Hymenolepis species, the absence of an internal membrane is exclusive. Hymenolepis diminuta is an incorrect response. Hymenolepis ova, although similar in size and shape with those of Taenia species, have a thin non-striated outer shell and a distinctive inner membrane enclosing the three pairs of hooklets. Diphyllobothrium latum is an incorrect response. Diphyllobothrium ova are large (up to 70 µm), have a smooth shell with a distinctive, inconspicuous non-shouldered operculum at one end. Internal cleavage that is devoid of hooklets extends to the inner shell membrane. Dipylidium caninum is an incorrect response. Dipylidium ova are small, have a thin smooth shell, and are spherical and arranged in packets within the ovary of the adult worm. An internal membrane and hooklets are absent.

An expectorated sputum specimen is received for culture at your laboratory. You are preparing to make slides and inoculate the media and are observing the gross appearance of the specimen. The specimen is a mixture of watery, bloody, and purulent material. Which area(s) of the specimen is most likely indicated for culture and slides? 1) Any area is acceptable for culture and slide preparation. 2) Only the watery portion of the specimen is most indicated for culture and slide preparation. 3) The bloody and purulent areas of the specimen are most indicated for culture and slide preparation. 4) An equal sampling of the watery, bloody, and purulent areas are most indicated for culture and slide preparation.

The bloody and purulent areas of the specimen are most indicated for culture and slide preparation When receiving an expectorated sputum specimen, one must be careful to select the appropriate area of the specimen. The bloody and purulent areas of the specimen are most indicated for culture and slide preparation. The watery area is most likely contaminated with upper respiratory flora and must be avoided. The slides will be used to screen the quality of the specimen and any specimen containing more than 10 squamous epithelial cells per low-power field (100X) should be rejected for culture.

When serotyping Salmonella, Shigella, and Escherichia coli 0157:H7, typing should be performed from a non-sugar-containing medium, such as 5% sheep blood agar. The reason for this is which of the following? 1) To prevent false positive results 2) To prevent false negative results 3) To prevent both false positive and false negative results 4) It does not matter what type of media as there is no interference in serotyping

To prevent false positive results To prevent false positive results is the correct answer because sugar containing media, such as MacConkey and Triple Sugar Iron agar, can cause the organism to autoagglutinate and give false positive serotyping results. To prevent false negative results is incorrect because false positive serotyping results occur not false negative. To prevent both false positive and false negative results is incorrect because only false positive serotyping results occur not false negative serotyping results. It does not matter what type of media as there is no interference in serotyping is incorrect because the type of media does matter due to autoagglutination of organisms grown on glucose containing media.

The scaling, pink-white appearance of the foot shown in the photograph is characteristic of "Athlete's Foot", a skin infection incurred by athletes after barefoot walks to and from the shower stall. A cottony gray-white fungal colony grew in 5 days from a skin scraping. The identification was made by the microscopic appearance of a mount made from the colony as illustrated in the photomicrograph. From these observations, select the name of the fungal species. 1) Microsporum gypseum 2) Trichophyton tonsurans 3) Trichophyton rubrum 4) Epidermophyton floccosum

Trichophyton rubrum Trichophyton rubrum is the correct response. The tinea pedis ("athlete's foot") skin lesion as observed in this exercise may be caused by select dermatophyte species from direct contact with spore-contaminated soil or water. The microscopic appearance of delicate hyphae from the sides of which are produced teardrop-shaped microconidia in a "birds on a fence" arrangement is characteristic of T. rubrum, the dermatophyte most commonly associated with this type of infection. Microsporum gypseum produces large, multi-segmented macroconidia with rounded ends produced from conidiophores borne laterally from nonbranching septate hyphae. Trichophyton tonsurans produces a mixture of club-shaped and spherical microconidia varying in size, but borne laterally and irregularly from nonbranching septate hyphae and not in the "birds on a fence" arrangement. Epidermophyton floccosum produces large club-shaped macroconidia, each with 3 - 5 cells separated by transverse septa, arranged in loose clusters rather than as "birds on a fence". Microconidia are not produced.

The upper image to the right depicts a small circular white colony with a downy surface, recovered from a skin scraping specimen after 7 days of growth on agar supplemented with thiamine and inositol. The identification is made by observing the aerial hyphae (middle image) and the underlying chains of pigmented chlamydospores among the hyphae (lower image). Based on this information, what is the most likely identification of this dermatophyte? 1) Microsporum gypseum 2) Trichophyton tonsurans 3) Trichophyton verrucosum 4) Epidermophyton floccosum

Trichophyton verrucosum Trichophyton verrucosum exhibits a small-growing white downy colony which is not distinctive. The indication that the colony grew on agar containing thiamine and inositol may be a clue that one of the Trichophyton species, particularly T. verrucosum was suggested. The antler-type hyphae and the chains of darkly pigmented chlamydospores as observed along the underlying hyphae indeed are the microscopic features for T. verrucosum. Microsporum gypseum colonies are more rapidly growing, maturing in 3 - 5 days, and have a more sugary or granular surface as conidia are produced. Microscopically, large and multi-segmented macroconidia with rounded ends are produced from conidiophores borne laterally from nonbranching septate hyphae. Triphophyton tonsurans colonies are also slow growing, but have a granular buff-colored surface with prominent radial rugae. Microscopically, club-shaped and spherical conidia are produced laterally from nonbranching septate hyphae. Chlamydospores are not produced. Epidermophyton floccosum colonies are more rapidly growing, maturing in 3 - 5 days, and have a khaki-green pigment with peripheral radiating hyphal strands when mature. Microscopically, large club-shaped macroconidia with 3 - 5 cells are characteristic. Neither microconidia nor chlamydospores are produced.

A cerebrospinal (CSF) cell count is 1000 cells/µL. The cells on differential are predominantly lymphocytes. CSF protein is moderately elevated and the glucose is normal. No microorganisms were observed on a Gram stain smear of the CSF sediment. What is the MOST likely condition? 1) Bacterial meningitis 2) Viral meningitis 3) Normal spinal fluid 4) Tubercular meningitis

Viral meningitis The correct response is viral meningitis which is characterized by an elevated white blood cell count in the range of 10-2000/µL and a predominance of lymphocytes on the white cell differential. The glucose is within reference range and the total protein is slightly to moderately elevated. Bacterial meningitis is characterized by an elevated white blood cell count in the range of 5-20,000/µL with a predominance of neutrophils on the white cell differential. The glucose is markedly decreased (<45 mg/dL) while the total protein is markedly elevated (>100 mg/dL). In normal spinal fluid, 0-5 WBC/µL are seen with a total protein of 15-50 mg/dL and glucose in the range of 45-100 mg/dL. CSF glucose is evaluated in relation to the serum glucose level; the CSF glucose is normally 60-70% of the blood glucose. In tubercular and fungal meningitis, the WBC ranges from 5-2000/µL with a predominance of mononuclear (lymphocytes and monocytes) cells. The protein is moderately elevated to a value of over 50 mg/dL and the glucose is either normal or slightly decreased.

A 27-year-old man who had recently worked as a medical laboratory scientist on the Navajo Native American reservation presented to the emergency department with high fever, diarrhea, and prostration. Axillary lymph nodes were hemorrhagic and enlarged. A smear was prepared from a lymph node aspirate and many Gram-negative bacilli were noted. The bacilli demonstrated a marked bipolar staining reaction described as a "safety-pin appearance" with Wayson stain. What is the most likely identification of this organism? 1) Yersinia pestis 2) Streptobacillus moniliformis 3) Capnocytophaga 4) Brucella melitensis

Yersinia pestis Yersinia pestis, the cause of plague, is the correct answer. Plague is a zoonosis that is transmitted through the flea-rodent life cycle and is diagnosed through demonstration of the organism in blood cultures and lymph nodes. The bipolar Gram-negative bacilli stained by the Wayson method found in aspirates of lymph nodes as well as the patient's history provide strong evidence for Yersinia. Y. pestis is found in the Western United States where rodents and cats carry infected fleas. The other choices do not have geographic or clinical symptoms consistent with Y. pestis. Streptobacillus moniliformis is a filamentous or pleomorphic Gram-negative bacillus found as normal flora in the oropharynx of rodents. It is the cause of Rat Bite Fever which occurs from the bite or the scratch of an infected rat. There are nonspecific signs of infection which include fever, chills, and muscle aches followed by joint pain and a skin rash on the palms of the hands and soles of the feet. The affected populations include children, laboratory technicians, and pet store workers. S. moniliformis is also the cause of Haverhill fever, which is acquired through the ingestion of milk contaminated with the organism. Capnocytophaga are Gram-negative bacilli that are found as normal flora of dogs and may cause human infection following dog bites, which can range from a localized infection to bacteremia. Brucella melitensis is a Gram-negative bacillus found as normal flora in goats and sheep. Humans acquire B. melitensis infections through ingestion of contaminated animal products, including milk; direct contact through skin abrasions, and through inhalation of contaminated aerosols. Brucella is most often isolated from the blood in cases of brucellosis, also known as Undulant Fever.


संबंधित स्टडी सेट्स

Salesforce Business Analyst Certification

View Set

ANAT CH 21: LYMPHATIC AND IMMUNE SYSTEM

View Set

Salesforce Identity and Access Management Architect

View Set

Maternal-newborn Ch. 23 Conditions Occurring after Delivery

View Set

PHI -186 - Chapter 4 - The Nature of Capitalism

View Set

ENG 102 EXAM 2 Poetry Mrs. Towels

View Set

Word 2013 Using Advanced Options 1.14 review

View Set